Histopathology Flashcards

1
Q
  1. Cardiovascular pathology
A Monckeberg arteriosclerosis
B Infective endocarditis
C Dressler’s syndrome
D Dilated cardiomyopathy
E Rheumatic heart disease
F Left heart failure
G Hypertrophic obstructive cardiomyopathy
H Aortic stenosis
I Carcinoid syndrome

1 A 36-year-old man presents to accident and emergency with a 1 day history of a fever of 39.2 C and night sweats. A new heart murmur is detected by the on-call cardiologist. The patient admits to being an intravenous drug user.

A

1) B

Infective endocarditis (IE; B) results from bacterial-vegetation of heart valves. Acute IE has a time course of days and is usually caused by Staphylococcus aureus in intravenous drug users; both sides of the heart can be affected, but the right heart is most commonly affected, because the lungs filter out many organisms, so that the left side of the heart gets less exposure to organisms. Subacute IE has a time course of weeks/months and is generally secondary to Streptococcus viridans infection after dental procedures; only abnormal valves are affected and hence these are more likely to be on the left side of the heart because those valves are more commonly damaged as they are on the high pressure side of the heart. Perforation of the valve leaflets and rupture of papillary muscles may lead to aortic or mitral regurgitation.

Monckeberg arteriosclerosis (A) involves focal calcification of the media of small medium-sized arteries. It usually presents in patients over 50 years of age and unlike atherosclerosis, there is no associated inflammation in the pathogenesis.

Dilated cardiomyopathy (D) occurs due to a progressive loss of myocytes resulting in a left ventricular ejection fraction of less than 40%. Causes include alcohol, chemotherapy (e.g. doxorubicin) and viral myocarditis.

Hypertrophic obstructive cardiomyopathy (G) is an autosomal dominant condition caused by a mutation in the Beta-myosin heavy chain. Histological features include myocyte hypertrophy and disarray.

Carcinoid syndrome (I) occurs due to 5-hydroxyindoleacetic acid producing tumours and is characterized by episodic flushing, abdominal cramps and diarrhoea. Right sided valve abnormalities may also result.

How well did you know this?
1
Not at all
2
3
4
5
Perfectly
2
Q
  1. Cardiovascular pathology
A Monckeberg arteriosclerosis
B Infective endocarditis
C Dressler’s syndrome
D Dilated cardiomyopathy
E Rheumatic heart disease
F Left heart failure
G Hypertrophic obstructive cardiomyopathy
H Aortic stenosis
I Carcinoid syndrome

2 A 64-year-old man presents to accident and emergency due to a collapse at home. An ejection systolic murmur is heard at the upper-left sternal edge.

A

2) H

Aortic stenosis (H) occurs when there is an opening defect in the aortic valve. Causes include age-related degenerative calcification, rheumatic heart disease and congenital malformations (bicuspid valve). Calcification is confined to the cusps. Clinical presentation includes syncope, angina and dyspnoea. On examination an ejection systolic murmur, narrow pulse pressure and/or slow rising pulse may be detected. If due to a bicuspid valve an ejection systolic click may be heard. LVH may develop as a consequence of chronic pressure overload.

Monckeberg arteriosclerosis (A) involves focal calcification of the media of small medium-sized arteries. It usually presents in patients over 50 years of age and unlike atherosclerosis, there is no associated inflammation in the pathogenesis.

Dilated cardiomyopathy (D) occurs due to a progressive loss of myocytes resulting in a left ventricular ejection fraction of less than 40%. Causes include alcohol, chemotherapy (e.g. doxorubicin) and viral myocarditis.

Hypertrophic obstructive cardiomyopathy (G) is an autosomal dominant condition caused by a mutation in the Beta-myosin heavy chain. Histological features include myocyte hypertrophy and disarray.

Carcinoid syndrome (I) occurs due to 5-hydroxyindoleacetic acid producing tumours and is characterized by episodic flushing, abdominal cramps and diarrhoea. Right sided valve abnormalities may also result.

How well did you know this?
1
Not at all
2
3
4
5
Perfectly
3
Q
  1. Cardiovascular pathology
A Monckeberg arteriosclerosis
B Infective endocarditis
C Dressler’s syndrome
D Dilated cardiomyopathy
E Rheumatic heart disease
F Left heart failure
G Hypertrophic obstructive cardiomyopathy
H Aortic stenosis
I Carcinoid syndrome

3 A widowed 72-year-old woman who has passed away at home is sent for autopsy due to unknown cause of death. Post-mortem examination reveals a nutmeg liver and haemosiderin-laden macrophages in the lungs.

A

3) F

Left heart failure (F) results in the inability of the heart to meet the demands of the body. It is either due to increased demand (high output failure) or reduced supply (low output failure) of blood. Causes of high output failure include severe anaemia and hyperthyroidism, while low output failure occurs due to ischaemic heart disease, hypertension and aortic/mitral valve defects. Clinical features include dyspnoea, orthopnoea and paroxysmal nocturnal dyspnoea. Histological findings include dilated ventricles, thin walls, nutmeg liver and haemosiderin macrophages in the lungs.

Monckeberg arteriosclerosis (A) involves focal calcification of the media of small medium-sized arteries. It usually presents in patients over 50 years of age and unlike atherosclerosis, there is no associated inflammation in the pathogenesis.

Dilated cardiomyopathy (D) occurs due to a progressive loss of myocytes resulting in a left ventricular ejection fraction of less than 40%. Causes include alcohol, chemotherapy (e.g. doxorubicin) and viral myocarditis.

Hypertrophic obstructive cardiomyopathy (G) is an autosomal dominant condition caused by a mutation in the Beta-myosin heavy chain. Histological features include myocyte hypertrophy and disarray.

Carcinoid syndrome (I) occurs due to 5-hydroxyindoleacetic acid producing tumours and is characterized by episodic flushing, abdominal cramps and diarrhoea. Right sided valve abnormalities may also result.

How well did you know this?
1
Not at all
2
3
4
5
Perfectly
4
Q
  1. Cardiovascular pathology
A Monckeberg arteriosclerosis
B Infective endocarditis
C Dressler’s syndrome
D Dilated cardiomyopathy
E Rheumatic heart disease
F Left heart failure
G Hypertrophic obstructive cardiomyopathy
H Aortic stenosis
I Carcinoid syndrome

4 A 54-year-old man presents to accident and emergency with fever and pleuritic chest pain. It is noted that the patient suffered a myocardial infarction 4 weeks previously.

A

4) C

Dressler’s syndrome (C) is an autoimmune complication of myocardial infarction (MI) that occurs approximately 4 weeks after the episode. It is characterized by chest pain, fever and a pericardial rub. The complications of MI can be classified according to how they present temporally. Complications of MI that may occur within 1 week include arrhythmias (most commonly ventricular fibrillation and ventricular tachycardia), myocardial rupture, valve incompetence (causing regurgitation) and cardiogenic shock. Later developments include ventricular aneurysm, pericarditis and the aforementioned Dressler’s syndrome.

Monckeberg arteriosclerosis (A) involves focal calcification of the media of small medium-sized arteries. It usually presents in patients over 50 years of age and unlike atherosclerosis, there is no associated inflammation in the pathogenesis.

Dilated cardiomyopathy (D) occurs due to a progressive loss of myocytes resulting in a left ventricular ejection fraction of less than 40%. Causes include alcohol, chemotherapy (e.g. doxorubicin) and viral myocarditis.

Hypertrophic obstructive cardiomyopathy (G) is an autosomal dominant condition caused by a mutation in the Beta-myosin heavy chain. Histological features include myocyte hypertrophy and disarray.

Carcinoid syndrome (I) occurs due to 5-hydroxyindoleacetic acid producing tumours and is characterized by episodic flushing, abdominal cramps and diarrhoea. Right sided valve abnormalities may also result.

How well did you know this?
1
Not at all
2
3
4
5
Perfectly
5
Q
  1. Cardiovascular pathology
A Monckeberg arteriosclerosis
B Infective endocarditis
C Dressler’s syndrome
D Dilated cardiomyopathy
E Rheumatic heart disease
F Left heart failure
G Hypertrophic obstructive cardiomyopathy
H Aortic stenosis
I Carcinoid syndrome

5 A 46-year-old man is referred to the cardiology outpatient clinic. On investigation he is found to have mitral regurgitation and has a past history of St Vitus Dance when he was in school and a mild pericarditis.

A

5) E

Rheumatic heart disease (E) is an inflammatory condition most commonly affecting the connective tissue of the heart (but also joints and central nervous system). It occurs several weeks after throat infection with group A Beta-haemolytic streptococci usually under the age of 10 years. Cardiac complications include endocarditis (causing verroucous lesions of the heart valves); myocarditis (containing Aschkoff-bodies and Anitschow cells causing dilatation of the mitral ring, hence mitral regurgitation); pericarditis (fibrous exudate causing friction rub). Any layer of the heart can be affected, potentially leading to pancarditis. Many years after recovery from acute rheumatic fever, chronic rheumatic heart disease occurs, with fibrosis of the mitral and aortic valves that can occur. The history of St Vitus Dance Suggests Sydenham’s chorea, a well known feature of acute rheumatic fever.

Monckeberg arteriosclerosis (A) involves focal calcification of the media of small medium-sized arteries. It usually presents in patients over 50 years of age and unlike atherosclerosis, there is no associated inflammation in the pathogenesis.

Dilated cardiomyopathy (D) occurs due to a progressive loss of myocytes resulting in a left ventricular ejection fraction of less than 40%. Causes include alcohol, chemotherapy (e.g. doxorubicin) and viral myocarditis.

Hypertrophic obstructive cardiomyopathy (G) is an autosomal dominant condition caused by a mutation in the Beta-myosin heavy chain. Histological features include myocyte hypertrophy and disarray.

Carcinoid syndrome (I) occurs due to 5-hydroxyindoleacetic acid producing tumours and is characterized by episodic flushing, abdominal cramps and diarrhoea. Right sided valve abnormalities may also result.

How well did you know this?
1
Not at all
2
3
4
5
Perfectly
6
Q
  1. Mitral valve vegetation
    65yo patient with advanced breast malignancy and a hx of multiple systemic emboli suffers a stroke. O/E: no cardiac murmurs but an echo reveals small bland vegetations on the mitral valve. Blood cultures are negative. What is the most likely diagnosis?
A Infective endocarditis
B Acute rheumatic fever
C Non-bacterial thrombotic endocarditis
D Chronic rheumatic valvular disease
E Libman–Sacks endocarditis
A

C

Non-bacterial thrombotic endocarditis (NBTE; C) commonly affects patients over 40 years of age and is often characterized by the absence of inflammation or bacteria. Sterile fibrin and platelet vegetations are present on cardiac valves, more commonly affecting left-sided heart valves (mitral > aortic) and are also associated with numerous diseases, especially advanced stage malignancy. NBTE is a source of thrombo-embolism to the brain, heart, kidneys, and recurrent emboli are a hallmark feature.

Infective endocarditis (A) is unlikely in this case because of the absence of a persistent fever, no new or changing murmur and negative blood cultures. Other absent features of infective endocarditis include Roth’s spots, Janeway lesions and Osler’s nodes.

Acute rheumatic fever (B), mainly affecting children, commonly involves the endocardium and typically results in verruca formation on the left-sided heart valves. Vegetations are small as described, but they are much more likely to cause stenosis and are therefore inconsistent with this patient.

Chronic rheumatic valvular disease (D), mainly affecting adults, is a sequela of earlier rheumatic fever and causes thickening of valve leaflets, especially along lines of closure, and thickening, shortening and fusion of the chordate tendineae.

Libman–Sacks endocarditis (E) is very commonly assoc with SLE and rarely causes emboli.

How well did you know this?
1
Not at all
2
3
4
5
Perfectly
7
Q
  1. Bone pathology
A Osteoporosis
B Fibrous dysplasia
C Paget’s disease
D Osteomalacia
E Osteochondroma
F Osteoid osteoma
G Renal osteodytrophy 
H Enchondroma
I Giant cell tumour

1 A 35-year-old man with pain and difficulty bending his left knee. X-ray reveals many lytic lesions in the epiphysis of the patient’s left knee.

A

1) I

Giant cell tumour (GCT; I) is a borderline malignant tumour of giant osteoclast cells. The cells are similar to those found in Paget’s disease as they have multiple nuclei (>20). The osteoclastic cells cause lytic lesions in the epiphyses (especially around the knee) that are visible on X-ray and may give a characteristic ‘soap bubble’ appearance. Histological features include multinucleated giant osteoclasts with surrounding ovoid and spindle cells.

Osteochondroma (E) is a bone neoplasm affecting those aged between 10 and 30 years. It targets the large joints and is a low malignancy risk.

Osteoid osteoma (F) is a painful bone neoplasm. X-ray investigation reveals central nidus (luscent) with sclerotic rim (opaque) pattern.

Renal osteodystrophy (G) is also known as secondary hyperparathyroidism. It occurs in patients with renal failure due to build-up of phosphate, damage to 1alpha-hydroxylase, osteosclerosis and aluminium toxicity.

Enchondroma (H) is a bone neoplasm most prevalent in patients aged between 10 and 40 years. It affects the small joints and has a high risk of malignant transformation.

How well did you know this?
1
Not at all
2
3
4
5
Perfectly
8
Q
  1. Bone pathology
A Osteoporosis
B Fibrous dysplasia
C Paget’s disease
D Osteomalacia
E Osteochondroma
F Osteoid osteoma
G Renal osteodytrophy 
H Enchondroma
I Giant cell tumour

2 A 38-year-old woman presents to her GP with generalized bone pain. X-ray reveals areas of pseudofracture, especially in the ribs.

A

2) D

Osteomalacia (D) is defined as the insufficient mineralization of bone due to vitamin D deficiency. Reduced intake, malabsorptive conditions, phenytoin and chronic liver disease can cause vitamin D deficiency. A low vitamin D level causes hypocalcaemia resulting in increased PTH release (normalizing calcium). PTH stimulates osteoclastic activity causing bones to be soft and epiphyses to widen. Clinical features include craniotabes, bone pain, proximal weakness and pseudo-fractures (looser zones). Craniotabes is the descriptive term for the soft and elastic occipitoparietal bones causing an elastic recoil sensation when pushed.

Osteochondroma (E) is a bone neoplasm affecting those aged between 10 and 30 years. It targets the large joints and is a low malignancy risk.

Osteoid osteoma (F) is a painful bone neoplasm. X-ray investigation reveals central nidus (luscent) with sclerotic rim (opaque) pattern.

Renal osteodystrophy (G) is also known as secondary hyperparathyroidism. It occurs in patients with renal failure due to build-up of phosphate, damage to 1alpha-hydroxylase, osteosclerosis and aluminium toxicity.

Enchondroma (H) is a bone neoplasm most prevalent in patients aged between 10 and 40 years. It affects the small joints and has a high risk of malignant transformation.

How well did you know this?
1
Not at all
2
3
4
5
Perfectly
9
Q
  1. Bone pathology
A Osteoporosis
B Fibrous dysplasia
C Paget’s disease
D Osteomalacia
E Osteochondroma
F Osteoid osteoma
G Renal osteodytrophy 
H Enchondroma
I Giant cell tumour

3 A 65-year-old woman is referred to the rheumatologist after suffering recurrent falls. Blood tests are all unremarkable but a DEXA scan reveals a T-score of 2.8.

A

3) A

Osteoporosis (A) is defined by reduced bone density (reduced quantity) but normal quality. Reduced circulating oestrogen concentration causes IL-1 and IL-6 levels to rise causing osteoclastic activity. Osteoporosis primarily affects the vertebrae and hips. Primary osteoporosis occurs in post-menopausal women. Secondary causes include lifestyle choices (smoking, alcohol, inactivity), drugs (steroid, goserelin), low BMI as well as thyroid and parathyroid disease. Diagnosis is made using a DEXA scan; a T-score of <1 reflects osteopenia and a score of <2.5 suggests osteoporosis. All blood markers of metabolic bone disease are normal.

Osteochondroma (E) is a bone neoplasm affecting those aged between 10 and 30 years. It targets the large joints and is a low malignancy risk.

Osteoid osteoma (F) is a painful bone neoplasm. X-ray investigation reveals central nidus (luscent) with sclerotic rim (opaque) pattern.

Renal osteodystrophy (G) is also known as secondary hyperparathyroidism. It occurs in patients with renal failure due to build-up of phosphate, damage to 1alpha-hydroxylase, osteosclerosis and aluminium toxicity.

Enchondroma (H) is a bone neoplasm most prevalent in patients aged between 10 and 40 years. It affects the small joints and has a high risk of malignant transformation.

How well did you know this?
1
Not at all
2
3
4
5
Perfectly
10
Q
  1. Bone pathology
A Osteoporosis
B Fibrous dysplasia
C Paget’s disease
D Osteomalacia
E Osteochondroma
F Osteoid osteoma
G Renal osteodytrophy 
H Enchondroma
I Giant cell tumour

5 A 50-year-old man presents to his GP with pain in his arms and legs. The patient also complains of shooting pains down his left leg as well as worsening shortness of breath.

A

5) C

Paget’s disease (C) is a disease of bone remodelling whereby new bone is larger but weaker and prone to fracture. During the initial lytic phase giant osteoclasts with multiple nuclei rapidly resorb bone. In the mixed phase, osteoblast activity leads to increased bone mass. In the final osteosclerotic phase, bone formation continues but is woven and weak, with collagen arranged haphazardly resulting in a mosaic pattern. Complications can arise from deformities that cause impingement of nerves. Bone marrow infiltration of weak woven bone can lead to high output heart failure.

Osteochondroma (E) is a bone neoplasm affecting those aged between 10 and 30 years. It targets the large joints and is a low malignancy risk.

Osteoid osteoma (F) is a painful bone neoplasm. X-ray investigation reveals central nidus (luscent) with sclerotic rim (opaque) pattern.

Renal osteodystrophy (G) is also known as secondary hyperparathyroidism. It occurs in patients with renal failure due to build-up of phosphate, damage to 1alpha-hydroxylase, osteosclerosis and aluminium toxicity.

Enchondroma (H) is a bone neoplasm most prevalent in patients aged between 10 and 40 years. It affects the small joints and has a high risk of malignant transformation.

How well did you know this?
1
Not at all
2
3
4
5
Perfectly
11
Q
  1. Bone pathology
A Osteoporosis
B Fibrous dysplasia
C Paget’s disease
D Osteomalacia
E Osteochondroma
F Osteoid osteoma
G Renal osteodytrophy 
H Enchondroma
I Giant cell tumour

8yo boy has been diagnosed with precocious puberty. A routine examination by the paediatrician reveals café-au-lait spots on the child’s back. The boy has had numerous fractures of his femur and tibia bilaterally after falls.

A

4) B

Fibrous dysplasia (B) occurs due to the developmental arrest of normal bone structures secondary to an osteoblast maturation defect. The most common sites affected are the proximal femur and ribs. On X-ray, fibrous dysplasia may cause a ground-glass or soap bubble appearance. Histological investigation reveals trabeculae that lack osteoblastic rimming. Two possible syndromes can arise:

1) Mono-ostotic (70%) affecting femurs more than ribs occurring in patients under 30 years of age, and
2) McCune–Albright syndrome (30%) that is poly-ostotic and causes café-au-lait spots and precocious puberty.

Osteochondroma (E) is a bone neoplasm affecting those aged between 10 and 30 years. It targets the large joints and is a low malignancy risk.

Osteoid osteoma (F) is a painful bone neoplasm. X-ray investigation reveals central nidus (luscent) with sclerotic rim (opaque) pattern.

Renal osteodystrophy (G) is also known as secondary hyperparathyroidism. It occurs in patients with renal failure due to build-up of phosphate, damage to 1alpha-hydroxylase, osteosclerosis and aluminium toxicity.

Enchondroma (H) is a bone neoplasm most prevalent in patients aged between 10 and 40 years. It affects the small joints and has a high risk of malignant transformation.

How well did you know this?
1
Not at all
2
3
4
5
Perfectly
12
Q
  1. Gastritis
    A 38-year-old man is a known gastritis patient. The most recent endoscopy and biopsy has detected that the area most severely affected is the pyloric antrum. He also has susceptibility for developing a gastric MALT lymphoma in the future. What is the most likely diagnosis?

A Menetrier’s disease (hyperplastic hypersecretory gastropathy)
B Acute gastritis
C Helicobacter-associated chronic gastritis
D Autoimmune chronic gastritis
E Reactive/reflux chronic gastritis

A

C

Helicobacter pylori-associated gastritis (C) is the most common form of chronic gastritis, accounting for 90% of cases, and it is known that the pyloric antrum is the most severely affected area. An immune response is established and the infection may potentially persist for years. Around 75% of MALT (mucosa-associated lymphoid tissue) lymphoma or MALToma cases are associated with H. pylori infection. MALT is a system of small lymphoid tissue that regulates mucosal immunity and is present in a variety of organs in the body.

In addition to the infectious cause of chronic gastritis, there are 2 other forms. The autoimmune chronic gastritis (D) is associated with pernicious anaemia in the elderly and typically affects the body of the stomach.

In reactive gastritis (E) the dominant feature is the epithelial change with minimal inflammation that can be idiopathic or due to reflux of bile-containing duodenal fluid or drugs.

Acute gastritis (B) is often a superficial form resulting from ingested chemicals and hypotension, unlikely in this patient who appears to have a more chronic presentation.

Menetrier’s disease (A) is a rare disease characterized by gross hyperplasia of gastric pits and a marked increase in mucosal thickness and typically affects the fundus and body of the stomach.

How well did you know this?
1
Not at all
2
3
4
5
Perfectly
13
Q
  1. Diseases of the exocrine pancreas
    A 50-year-old known alcoholic man has persistent severe epigastric pain radiating to the back and has experienced weight loss of 5kg in 2 months. On initial presentation, the patient is not jaundiced. On contrast enhanced CT scan there are multiple calcific densities along the line of the main pancreatic duct. On histological examination, there is evidence of parenchymal fibrosis and large ducts containing insipissated secretions. What is the most likely diagnosis?
A Chronic pancreatitis
B Carcinoma in the head of the pancreas
C Diabetes mellitus type 2
D Acute pancreatitis
E Pseudocysts
A

A

Chronic pancreatitis (A) causes irreversible loss of function. Histology shows chronic inflammation with parenchymal fibrosis, loss of pancreatic parenchymal elements and duct strictures with formation of intrapancreatic calculi. Jaundice may occur; it is a presenting feature in only a small proportion of patients and would be secondary to common bile duct obstruction during its course through the fibrosed head of the pancreas. Grossly, the pancreas is replaced by firm fibrous tissue within which are dilated ducts and areas of calcification. The pathogenesis of acute pancreatitis (D) is similar but with a lack of permanent impairment and often resolves with supportive therapy. Histologically it would only show acute inflammation and necrotic changes.

Carcinoma in the head of the pancreas (B) presents early with obstructive jaundice, which this patient does not have, despite the history of weight loss and epigastric pain.

Diabetes mellitus type 2 (C) is a metabolic disorder, where patients may present with polyuria and polydipsia due to the chronic hyperglycaemia and not with severe epigastric pain.

Pseudocysts (E) are collections of fluid and necrotic inflammatory debris in the pancreas that may be associated with acute or chronic pancreatitis but often resolve spontaneously.

How well did you know this?
1
Not at all
2
3
4
5
Perfectly
14
Q
  1. Diseases of the endocrine pancreas
    A 22-year-old man presents with polyuria and polydipsia. His fasting plasma glucose is 7.3mmol/L. He is Glutamic Acid Decarboxylase (GAD) antibody positive. What is the most likely diagnosis?
A Diabetes mellitus type 1
B Diabetes insipidus
C Psychogenic polydipsia
D Diabetes mellitus type 2
E Zollinger–Ellison syndrome
A

A

DMT1 (A) is an autoimmune disorder of adolescent onset that is characterized by antibody-mediated destruction of beta-cells of the islets of Langerhans. 90-95% of patients are HLA DR3 and HLA DR4 positive. T1DM can present with polyuria or with DKA, if the polyuria is ignored. The peak incidence of T1DM is 12–14 years, so the patient being young also makes this T1DM.

DMT2 (D) is not an autoimmune disorder and has no HLA associations and the patient usually is a bit older. T2DM used to be called “maturity asset diabetes”, as it commonly affected people >50 years old, although with the increase in obesity, patients are beginning to present with T2DM at a younger age, and in some patients, further tests are required before one can easily distinguish T1DM from T2DM. Patients with T1DM are GAD (glutamic Acid Decarboxylase Autoantibodies test) antibody positive, and this test has superceded the HLA tissue type, which is not diagnostic. The diagnosis of diabetes mellitus is on the finding of hyperglycaemia. Diagnosis of DM = fasting venous plasma glucose levels of >7.0mmol/L or a random venous plasma glucose level of >11.1mmol/L.

Diabetes Insipidus (B) is a disorder of the pituitary where there is either a failure of ADH production (cranial DI) or the renal distal tubules are refractory to the water reabsorptiveaction of ADH (nephrogenic DI). While both present with polyuria and polydipsia, DI is not related to plasma glucose levels.

Psychogenic polydipsia (C) is excessive fluid intake with no organic pathology. Described in schizophrenics and young children with emotional difficulties.

Zollinger–Ellison syndrome (E) is where there is gastric hypersecretion, multiple peptic ulcers and diarrhoea caused by gastric-secreting tumour of the pancreatic G cells. This may be part of MEN1 syndrome, with adenomas present in other endocrine glands.

How well did you know this?
1
Not at all
2
3
4
5
Perfectly
15
Q
  1. Viral hepatitis
    A 37-year-old man, while abroad, was involved in a road traffic accident and required a blood transfusion. He had an episode of acute hepatitis with the contraction of a DNA virus of the Hepadna group. There is a small chance this may progress to chronic hepatitis. What is the most likely viral hepatitis type?
A Hepatitis A
B Hepatitis B
C Hepatitis C
D Hepatitis D
E Hepatitis E
A

B

Hepatitis B (B) is a DNA virus of the Hepadna group. Transmission is commonly blood-borne but these can also be sexual and vertical transmission from mother to child. Most commonly, the infection can be asymptomatic with complete recovery, but also patients can develop acute or chronic hepatitis B infection. These patients always have a risk of developing chronic hepatitis.

Hepatitis A (A) infection never causes chronic hepatitis.

Hepatitis C (C) is an RNA flavivirus that also shows predominantly blood-borne spread and is very common among intravenous drug abusers. A high percentage (70%) will develop chronic hepatitis, increasing the life-long risk of cirrhosis and hepatocellular carcinoma.

Hepatitis D (D) is an RNA virus that is incomplete and can only cause infection in the presence of hepatitis B virus.

Hepatitis E (E) is an RNA virus that behaves in a similar fashion in terms of transmission and clinical features to hepatitis A virus.

How well did you know this?
1
Not at all
2
3
4
5
Perfectly
16
Q
  1. Anti-mitochondrial antibodies
    A 42-year-old woman, who has a history of joint and skin symptoms, presents with jaundice. Anti-mitochondrial antibodies are present and histologically there is evidence of a progressive, chronic granulomatous inflammation of the bile duct. What is the most likely diagnosis?
A Primary sclerosing cholangitis
B Autoimmune hepatitis
C Primary biliary cirrhosis
D alpha-1 Antitrypsin deficiency
E Alcoholic liver disease
A

C
Primary biliary cirrhosis (C) is the destruction of the intrahepatic bile duct, often assoc w. an immune component, with anti-mitochondrial antibodies present in 90% of cases and assoc w. other autoimmune diseases such as RA or scleroderma.

Primary sclerosing cholangitis (PSC) (A) also presents with obstructive jaundice and probably has an autoimmune element to it, but is very unlikely to show any autoantibodies. 60% cases assoc with UC.

Autoimmune hepatitis (B) is similar histologically to chronic viral hepatitis, and is associated with hyperglobulinaemia and anti-smooth muscle antibodies.

Alpha-1 antitrypsin deficiency (D) is an inherited condition, where individuals fail to produce protease inhibitor alpha-1 antitrypsin. This can cause hepatitis in adults due to the accumulation of abnormal alpha-1 antitrypsin globules in the liver.

Alcoholic liver disease (E) = history of alcohol abuse and the ethanol toxicity damages hepatocytes leading to fatty liver, acute hepatitis and cirrhosis.

How well did you know this?
1
Not at all
2
3
4
5
Perfectly
17
Q
  1. Congenital metabolic disorders
    A 23-year-old patient has an autosomal recessive disorder. The patient has demonstrated parkinsonian symptoms such as a hand tremor and has developed chronic hepatitis. On examination, he is found to have Kayser–Fleischer rings. Blood levels of serum ceruloplasmin are low. What is the most likely diagnosis?
A Wilson’s disease
B Genetic haemochromatosis
C alpha-1 Antitrypsin deficiency
D Reye’s syndrome
E Budd–Chiari syndrome
A

A

Wilson’s disease (A) is an autosomal recessive metabolic disorder that is caused by a mutation in the copper transport ATPase gene on chromosome 13. It results in failure of the liver to secrete the copper–ceruloplasmin complex into the plasma. This creates an overspill of copper into the blood that typically causes liver disease, central nervous system disease resembling Parkinson’s disease and the characteristic development of brown discolouration around the cornea (Kayser–Fleischer rings).

Genetic haemochromatosis (B) is excessive deposition of iron in tissues due to increased absorption of iron from the gut. Iron accumulates as haemosiderin in various organs giving them a rusty brown appearance, including hyperpigmentation of the skin. The diagnosis is based on the high saturation of transferrin in the blood, high serum iron and ferritin levels.

Alpha-1 antitrypsin deficiency (C) is not known to affect the central nervous system, but may cause chronic hepatitis and cirrhosis due to accumulation of the abnormal alpha-1 antitrypsin globules in the liver.

Reye’s syndrome (D) often occurs following an upper respiratory tract infection and was thought to occur in children exposed to aspirin. It is rare and presents with acute encephalopathy secondary to severe impairment of liver function.

Budd–Chiari syndrome (E) is the rare condition that is characterized by the occlusion of the main hepatic vein due to either local compression or thrombosis, or is idiopathic.

How well did you know this?
1
Not at all
2
3
4
5
Perfectly
18
Q
  1. Gastrointestinal diseases in children
    An 8-year-old Down syndrome boy presents with constipation, distended abdomen, vomiting and overflow diarrhoea. The cause is believed to be absence of ganglion cells in the myenteric plexus causing the failure of the dilation of the distal colon. What is the most likely diagnosis?
A Stenosis
B Hirschsprung’s disease
C Atresia
D Intussusception
E Volvulus
A

B

Congenital aganglionic megacolon, also known as Hirschsprung’s disease (B), is believed to be due to the absence of ganglion cells in the myenteric plexus causing the failure of the dilation of the distal colon.
Macroscopically = narrowing of an abnormally innervated bowel segment yet dilation and muscular hypertrophy of the bowel segment proximal to this.
Microscopically = absence of normal myenteric and submucosal plexus ganglion cells. often presents in early childhood with symptoms of colonic obstruction. reported association with Down syndrome.

Stenosis (A) implies incomplete obstruction, while atresia (C) implies complete obstruction. Both are rare in the colon but more commonly found in the duodenum and small intestine.

Intussusception (D) - when one portion of the bowel invaginates into the adjoining segment, most commonly at the ileoceacal valve. Bleeding takes place due to the venous congestion of the invaginated portion, which this patient does not have signs of.

Volvulus (E) is a type of mechanical disorder, where there is complete twisting of a loop of bowel at mesenteric base causing obstruction and infarction and severe pain on presentation.

How well did you know this?
1
Not at all
2
3
4
5
Perfectly
19
Q
  1. Colitis
    A 25-year-old white man is experiencing bloody diarrhoea and mucous discharge. Macroscopic analysis shows abnormality in the colon and rectum only and is continuous with a normal bowel wall thickness. The pattern of inflammation is confined to the mucosa of the bowel wall and no evidence of granulomas exists. What is the most likely diagnosis?
A Crohn’s disease
B Ulcerative colitis
C Ischaemic colitis
D Pseudomembranous colitis
E Viral gastroenteritis
A

B

Ulcerative colitis (B) is inflammation affecting the rectum and colon only in a contiguous fashion. There are many extra-intestinal manifestations including arthritis, myositis, uveitis/iritis, erythema nodosum, pyoderma gangrenosum and primary sclerosing cholangitis.

Crohn’s disease (A), on the other hand, can affect any region of the gastrointestinal tract with ‘skip lesions’. It is also associated with transmural inflammation and non-caseating granulomas, which this patient does not have evidence of. Wall is thickened and described as ‘rubber-hose’ and ‘cobblestone mucosa’. Ulcers tend to be linear and the lumen is narrowed. Extra-intestinal manifestations are also common in Crohn’s disease.

Ischaemic colitis (C) is the most common vascular disorder of the intesti- nal tract and usually occurs in segments in ‘watershed zones’ such as the splenic flexure or the rectosigmoid flexure. It may be transmural and may lead to perforation.

Pseudomembranous colitis (D) is a type of bacterial enterocolitis caused by the protein exotoxins of Clostridium difficile. The necrosis of the colonic mucosa characteristically causes the formation of a pseudomembrane. Patients develop fever, abdominal pain and diarrhoea.

Viral gastroenteritis (E) is the most common in children and infants and typically presents with cramps, vomiting and fever but no blood in stools.

How well did you know this?
1
Not at all
2
3
4
5
Perfectly
20
Q
  1. Neoplastic disease of the intestine
    A 39-year-old man is diagnosed with a colon cancer proximal to the splenic flexure that is poorly differentiated and highly aggressive. There are no associated adenomata. It is an autosomal dominant condition that involves gene mutations of DNA mismatch repair genes. What is the most likely diagnosis?
A Familial adenomatous polyposis
B Gardner’s syndrome
C Colorectal carcinoma
D Hereditary non-polyposis colorectal cancer
E Hamartomatous polyps
A

D
Hereditary non-polyposis colorectal cancer (HNPCC) (D) is an uncommon autosomal dominant disease but the cancers are poorly differentiated and highly aggressive, therefore screening for identification of carriers for surveillance is necessary.

Familial adenomatous polyposis (FAP) (A) is also a rare autosomal dominant condition that is caused by a mutation in the FAP gene on chromosome 5. It is characterized by the presence of adenomata in the large bowel, which this patient did not have, yet also carries a 90 per cent risk of developing into carcinoma by the age of 45.

Gardner’s syndrome (B) is similar clinically, pathologically and aetiologically to FAP and also carries a high carcinoma risk. However, there are very distinct extra-intestinal manifestations of Gardner’s syndrome, including multiple osteomas of the skull and mandible, epidermoid cysts and desmoid tumours.

Colorectal carcinoma (C) of the sporadic type is inconsistent with the history and age of the patient, as it is more commonly present in patients over the age of 60.

Hamartomatous polyps (E) often present in childhood or adolescence as part of Peutz–Jeghers syndrome and these carry a small risk of carcinomas. Patients also tend to have pigmented lesions around the mouth that are characteristic of this condition.

How well did you know this?
1
Not at all
2
3
4
5
Perfectly
21
Q
  1. Hydrosalpinx
    A 25-year-old woman presents to clinic with an inability to conceive and a past history of Chlamydia trachomatis infection. On ultrasonography, she is diagnosed with hydrosalpinx. Hydrosalpinx is the most likely complication of which of the below options?
A Endometriosis
B Adenomyosis
C Cervical intraepithelial neoplasia
D Salpingitis
E Human papillomavirus
A
D
Salpingitis (pelvic inflammatory disease; D) is inflammation of the fal- lopian tubes that is almost always caused by infection, in particular sexually transmitted infections including chlamydia, mycoplasma and gonococcus. Other related infections, such as an actinomyces infection, are associated with intrauterine contraceptive device use. Hydrosalpinx, a complication of salpingitis, is the dilation of the fallopian tube that is thin-walled and contains clear fluid. This is believed to be a sequel to previous inflammatory damage to the tube. The scarring sequelae are believed to include plical fusion, adhesions to the ovary, tubo-ovarian abscess, peritonitis, hydrosalpinx, infertility and ectopic pregnancy.

Endometriosis (A) occurs when endometrial glands develop outside the uterus and is associated with development of fibrous adhesions. It may develop in the Fallopian tube but will not cause a hydrosalpinx and is not often associated with a history of an infectious episode.

Adenomyosis (B) is a disorder of the uterus whereby the endometrium grows to develop deep within the myometrium.

Cervical intraepithelial neoplasia (C) is a pre-neoplastic (dysplastic) proliferation of epithelium of the transformation zone of the cervix. This is strongly associated with human papillomavirus (E) infection, particularly serotypes 16 and 18.

How well did you know this?
1
Not at all
2
3
4
5
Perfectly
22
Q
  1. Congenital causes of cardiovascular disease
    A 41-year-old man presents with severe central chest pain which he describes as ‘tearing’ in nature and radiating to the back. He is tall, with long limbs and long thin fingers. He also has an aortic regurgitation murmur. Histologically there is cystic medial necrosis in the aortic wall. In which syndrome are these findings most likely?
A Ortner’s syndrome
B Ehlers–Danlos syndrome
C Down syndrome
D Turner syndrome
E Marfan syndrome
A

E

Cystic medial necrosis is a disorder particularly affecting the aorta, causing focal degeneration of the elastic tissue and muscle fibres in the media, with accumulation of basophilic ground substance. This leads
to cyst-like pools between the fibres disrupting the normally parallel arrays. Clinically, aneurysm formation becomes more likely. It is more frequent after 40 years of age and is twice as common in males. There is evidence that links cystic medial necrosis to aortic dissection in patients with a variety of syndromes, the most common of which is Marfan’s syndrome (E). These patients are characteristically tall with long limbs and long thin fingers. All other syndromes are inconsistent with this patient.

The most common murmur in Ortner’s syndrome (A) is mitral stenosis, associated with an enlarged left atrium and recurrent laryngeal nerve palsy.

Turner’s syndrome (D) is only present in females. Congenital heart defects include: aortic coarctation, aortic stenosis, ventricular septal defect and atrial septal defect, but aortic dissection is uncommon.

In Ehlers–Danlos syndrome (B), cystic medial necrosis can cause fragile blood vessels but aortic regurgitation is not commonly present.

Down syndrome (C) patients may have congenital atrioventricular septal canal defects. Other causes of cystic medial necrosis in patients without Marfan’s syndrome are advanced age and chronic hypertension.

How well did you know this?
1
Not at all
2
3
4
5
Perfectly
23
Q
  1. Respiratory pathology
A Hyaline membrane disease
B Small cell carcinoma
C Extrinsic allergic alveolitis
D Bronchiectasis
E Non-small cell carcinoma
F Chronic bronchitis 
G Pulmonary oedema 
H Cystic fibrosis
I Sarcoidosis

1 A 40-year-old male presents to his GP with chronic cough with copious amounts of purulent mucus production. High resolution CT scans demonstrate dilated bronchi.

A

Bronchiectasis (D) is defined as the permanent dilatation of bronchi and bronchioles secondary to chronic inflammation. Causes are numerous, and include chronic pneumonia, for example due to Staphylococcus aureus or Haemophilus influenzae infection, obstructing tumours and cystic fibrosis. Histopathological findings include bronchial wall destruction and transmural inflammation. High-resolution computed tomography (CT) is the diagnostic modality of choice. Abscess formation, haemoptysis and pulmonary hypertension are complications that may arise as a result of bronchiectasis.

Hyaline membrane disease (A) is also known as respiratory distress syndrome. It occurs in premature neonates due to surfactant deficiency leading to hyaline deposition and hypoxia.

Extrinsic allergic alveolitis (C) occurs secondary to a type III hypersensitivity reaction. Long-term exposure to an inhaled allergen leads to pulmonary fibrosis.

Chronic bronchitis (F) is one end of the chronic obstructive pulmonary disease spectrum. Histological features include mucus gland hypertrophy, goblet cell hyperplasia/metaplasia and mucosal oedema.

Sarcoidosis (I) is a multisystem, non-caseating granulomatous disease. Inflammatory markers such as TNF-alpha , IFN-gamma and IL-12 play an important role in the inflammatory process.

How well did you know this?
1
Not at all
2
3
4
5
Perfectly
24
Q
  1. Respiratory pathology
A Hyaline membrane disease
B Small cell carcinoma
C Extrinsic allergic alveolitis
D Bronchiectasis
E Non-small cell carcinoma
F Chronic bronchitis 
G Pulmonary oedema 
H Cystic fibrosis
I Sarcoidosis

2 A 14-year-old girl is admitted to hospital after suffering her third bout of pneumonia caused by Pseudomonas aeruginosa infection. She also has a previous admission for pancreatitis.

A

2) H

Cystic fibrosis (CF; H) is an autosomal recessive condition caused by a mutation in the cystic fibrosis transmembrane conductance regulator (CFTR) protein that primarily affects the exocrine glands. There are several mutations responsible for CF, the most common being F508 mutation. Defective CFTR causes reduced secretion of chloride ions across epithelial cell membranes, resulting in increased sodium and hence water reabsorption into these cells. The result is viscous secretions from exocrine glands affecting multiple organs including the lungs (recurrent infections and bronchiectasis), gastrointestinal tract (distal intestinal obstruction syndrome) and pancreas (pancreatitis).

Hyaline membrane disease (A) is also known as respiratory distress syndrome. It occurs in premature neonates due to surfactant deficiency leading to hyaline deposition and hypoxia.

Extrinsic allergic alveolitis (C) occurs secondary to a type III hypersensitivity reaction. Long-term exposure to an inhaled allergen leads to pulmonary fibrosis.

Chronic bronchitis (F) is one end of the chronic obstructive pulmonary disease spectrum. Histological features include mucus gland hypertrophy, goblet cell hyperplasia/metaplasia and mucosal oedema.

Sarcoidosis (I) is a multisystem, non-caseating granulomatous disease. Inflammatory markers such as TNF-alpha , IFN-gamma and IL-12 play an important role in the inflammatory process.

How well did you know this?
1
Not at all
2
3
4
5
Perfectly
25
Q
  1. Respiratory pathology
A Hyaline membrane disease
B Small cell carcinoma
C Extrinsic allergic alveolitis
D Bronchiectasis
E Non-small cell carcinoma
F Chronic bronchitis 
G Pulmonary oedema 
H Cystic fibrosis
I Sarcoidosis

3 A 58-year-old man presents to his GP with haemoptysis and weight loss. He has a 30 pack–year history of smoking. He is referred to the oncologist for a biopsy, who determines ‘oat-shaped’ cells on microscopy.

A

3) B

Small cell carcinoma (B) is also known as ‘oat-cell’ carcinoma due to the appearance of the malignant cells under the microscope. They appear as nests of small round hyperchromatic cells that are fragile (chromatin smudging) and possess nuclear moulding. Small cell carcinomas are very aggressive with approximately 80% of cases having metastasized at the time of diagnosis. Small cell carcinomas also express neuroendocrine markers and can cause paraneoplastic syndromes such as Lambert–Eaton myasthenic syndrome. On chest X-rays, the cancer may be seen arising centrally.

Hyaline membrane disease (A) is also known as respiratory distress syndrome. It occurs in premature neonates due to surfactant deficiency leading to hyaline deposition and hypoxia.

Extrinsic allergic alveolitis (C) occurs secondary to a type III hypersensitivity reaction. Long-term exposure to an inhaled allergen leads to pulmonary fibrosis.

Chronic bronchitis (F) is one end of the chronic obstructive pulmonary disease spectrum. Histological features include mucus gland hypertrophy, goblet cell hyperplasia/metaplasia and mucosal oedema.

Sarcoidosis (I) is a multisystem, non-caseating granulomatous disease. Inflammatory markers such as TNF-alpha, IFN-gamma and IL-12 play an important role in the inflammatory process.

How well did you know this?
1
Not at all
2
3
4
5
Perfectly
26
Q
  1. Respiratory pathology
A Hyaline membrane disease
B Small cell carcinoma
C Extrinsic allergic alveolitis
D Bronchiectasis
E Non-small cell carcinoma
F Chronic bronchitis 
G Pulmonary oedema 
H Cystic fibrosis
I Sarcoidosis

4 A 62-year-old man presents to his GP with shortness of breath, lethargy and weight loss. The patient’s chest X-ray reveals a peripheral focal lesion in the left lung field.

A

4) E

Non-small cell carcinomas (E) comprise adenocarcinoma, squamous cell carcinoma and large cell carcinoma. Adenocarcinomas are gland forming and therefore will have mucin vacuoles within. This sub-type of non-small cell carcinoma may lead to atypical adenohyperplasia whereby atypical cells are seen to line the alveolar walls; hence adenocarcinoma is usually a peripheral lung cancer. Squamous cell carcinomas are histologically characterized by keratinization and intracellular ‘prickle’ desmosomes. Large cell carcinomas are undifferentiated forms of adenocarcinoma or squamous cell carcinoma.

Hyaline membrane disease (A) is also known as respiratory distress syndrome. It occurs in premature neonates due to surfactant deficiency leading to hyaline deposition and hypoxia.

Extrinsic allergic alveolitis (C) occurs secondary to a type III hypersensitivity reaction. Long-term exposure to an inhaled allergen leads to pulmonary fibrosis.

Chronic bronchitis (F) is one end of the chronic obstructive pulmonary disease spectrum. Histological features include mucus gland hypertrophy, goblet cell hyperplasia/metaplasia and mucosal oedema.

Sarcoidosis (I) is a multisystem, non-caseating granulomatous disease. Inflammatory markers such as TNF-alpha , IFN-gamma and IL-12 play an important role in the inflammatory process.

How well did you know this?
1
Not at all
2
3
4
5
Perfectly
27
Q
  1. Respiratory pathology
A Hyaline membrane disease
B Small cell carcinoma
C Extrinsic allergic alveolitis
D Bronchiectasis
E Non-small cell carcinoma
F Chronic bronchitis 
G Pulmonary oedema 
H Cystic fibrosis
I Sarcoidosis

5 A 53-year-old woman with a history of rheumatic fever presents to accident and emergency with severe shortness of breath, and has been coughing up pink frothy sputum for the past 2 days.

A

5) G

Pulmonary oedema (G) is defined as fluid collections in the alveoli which impairs gas exchange that can potentially lead to respiratory failure. Increased hydrostatic pressure causes of pulmonary oedema include heart failure, mitral stenosis, fluid overload and renal failure. Increased capillary permeability can also cause pulmonary oedema, for example due to pneumonia. Chest X-rays can distinguish between cardiac and non-cardiac causes of pulmonary oedema; the former will demonstrate alveolar oedema (bat’s wing appearance), Kerley B-lines, cardiomegaly, upper lobe diversion of blood vessels and effusions.

Hyaline membrane disease (A) is also known as respiratory distress syndrome. It occurs in premature neonates due to surfactant deficiency leading to hyaline deposition and hypoxia.

Extrinsic allergic alveolitis (C) occurs secondary to a type III hypersensitivity reaction. Long-term exposure to an inhaled allergen leads to pulmonary fibrosis.

Chronic bronchitis (F) is one end of the chronic obstructive pulmonary disease spectrum. Histological features include mucus gland hypertrophy, goblet cell hyperplasia/metaplasia and mucosal oedema.

Sarcoidosis (I) is a multisystem, non-caseating granulomatous disease. Inflammatory markers such as TNF-alpha , IFN-gamma and IL-12 play an important role in the inflammatory process.

How well did you know this?
1
Not at all
2
3
4
5
Perfectly
28
Q
  1. Gynaecological tumours
    A 42-year-old Afro-Caribbean woman is nulliparous and trying to conceive. She has been experiencing dysmenorrhoea. Ultrasound scan shows multiple rounded nodules within the myometrium. What is the most likely diagnosis?
A Cervical intraepithelial neoplasia
B Vulval carcinoma
C Leiomyoma
D Endometrial carcinoma
E CGIN (endocervical glandular dysplasia)
A

C

Leiomyoma (C), also called fibroids, is a benign smooth muscle tumour arising in the myometrium. They are the most common of all pelvic tumours, presenting often in women over 30 years of age and are more common in nulliparous and Afro-Caribbean women. The presentation often involves multiple large rounded nodules. They are well circumscribed with a pseudocapsule that may become pedunculated forming polyps.

Cervical intraepithelial neoplasia (A) is a dysplastic transformation at the cervix; it is associated with HPV infection and does not present as nodules in the myometrium, nor with dysmenorrhoea.

Vulval carcinoma (B) is often squamous and associated with HPV 16 infection and does not present with internal symptoms such as dysmenorrhoea.

Endometrial carcinoma (D) is malignant and more common in postmenopausal women. There is a hyperoestrogenic form and a non-hyperoestrogenic form, with superficial invasion of the myometrium and deep invasion respectively. The condition classically presents with postmenopausal bleeding and not dysmenorrhoea.

CGIN (E) is a rare disorder of the cervix that is difficult to diagnose and manage and may compromise fertility of the patient but does not involve the myometrium.

How well did you know this?
1
Not at all
2
3
4
5
Perfectly
29
Q
  1. Ovarian tumours
    A 20-year-old woman presents to accident and emergency with a distended abdomen resembling a pregnancy. She later develops acute onset of severe abdominal pain. An ultrasound identified a mass in her right ovary. Her abdomen is rigid and she is admitted for emergency surgery. It is believed that three embryonic germ cell layers are present. What is the most likely diagnosis?
A Teratoma of the ovary
B Serous tumour of the ovary
C Mucinous tumour of the ovary
D Endometrioid tumour of the ovary
E Clear cell carcinoma
A

A

Teratoma (A) is a common benign cyst that contains all three embry- onic germ cell layers and this is a torsion presentation which is the most common complication of teratomas. In mature cases there have been reports of teratomas containing features such as hair, teeth, bone and eyes. As they are encapsulated, teratomas are usually benign but may rarely undergo malignant change in postmenopausal women.

Serous, mucinous and endometrioid tumours (B, C, D) are epithelial ovarian tumours where serous tumours differentiate to mimic tubal epithelium, mucinous tumours differentiate to mimic endocervical or intestinal wall and endometrioid tumours mimic the endometrium. Pseudomyxoma peritonei, metastases from the appendix, are sometimes a characteristic feature of mucinous tumours.

Clear cell carcinomas (E) are uncommon and tend to be malignant with a poor prognosis. As the name suggest, they have a clear cytoplasm and are often associated with endometriosis.

How well did you know this?
1
Not at all
2
3
4
5
Perfectly
30
Q
  1. Nephritic disorders
A Minimal change disease
B Wegener’s granulomatosis
C Microscopic polyangitis
D Lupus nephritis
E IgA nephropathy
F Membranoproliferative glomerulonephritis
G Rapidly progressive glomerulonephritis
H Post-streptococcal glomerulonephritis
I Goodpasture’s syndrome

5 A 3-year-old boy is seen by the GP after his mother noticed swelling of his legs. A week previously the boy had been stung by a bee. Urine dipstick reveals the presence of proteinuria, while blood tests show hypoalbuminaemia and hyperlipidaemia.

A

5) A

Minimal change disease (A) is the most common cause of nephrotic syndrome in children. Triggers include a recent allergic reaction such as a bee sting (type I hypersensitivity reaction). Histological characteristics of renal biopsy specimens include a lack of structural change visible on light microscopy, while electron microscopy will demonstrate podocyte effacement. Steroids are the primary treatment modality, which lead to remission of disease in the vast majority of cases.

Microscopic polyangitis (C) is a small vessel vasculitis affecting the arterioles, venules and capillaries. This vasculitis is associated with focal necrotizing glomerulonephritis as well as the presence of perinuclear ANCA (p-ANCA) in the circulation directed towards cytoplasmic myeloperoxidase.

Lupus nephritis (D) is characterized by the deposition of IgG, IgM, IgA and C3 in the sub-endothelial segment of the glomerular basement membrane and in the mesangium.

Membranoproliferative glomerulonephritis (F) is defined by mesangial cell proliferation with thickening of the capillaries. Two types exist: type 1 in which there is classical and alternative complement pathway activation and type 2 that is associated with only alternative pathway activation.

Goodpasture’s syndrome (I) is characterized by the presence of anti-glomerular basement membrane proteins, specifically targeting type IV collagen. Pulmonary features include cough, dyspnoea and haemoptysis; renal features include haematuria, acute renal failure and nephrotic syndrome.

How well did you know this?
1
Not at all
2
3
4
5
Perfectly
31
Q
  1. Nephritic disorders
A Minimal change disease
B Wegener’s granulomatosis
C Microscopic polyangitis
D Lupus nephritis
E IgA nephropathy
F Membranoproliferative glomerulonephritis
G Rapidly progressive glomerulonephritis
H Post-streptococcal glomerulonephritis
I Goodpasture’s syndrome

4 A 65-year-old man with known renal failure is transferred to the renal team by the accident and emergency department with worsening renal function. A renal biopsy is taken which demonstrates the presence of crescents on histology; immunofluorescence staining of IgG/C3 reveals a granular pattern. The man is very ill with suggestions that he may require a renal transplant.

A

4) G

Rapidly progressive glomerulonephritis (RPGN; G) is the most aggressive of all glomerulonephritides, which may cause end-stage renal failure over a period of days. The three sub-types include immune complex disease, pauci-immune disease and anti-glomerular basement membrane disease, all of which demonstrate crescent formation on biopsy (proliferation of macrophages and parietal epithelial cells). Immunofluoresence of IgG/C3 distinguishes between the three sub-types: immune complex disease is characterized by granular staining, pauci-immune disease shows absent/scant staining, while anti-glomerular basement membrane disease demonstrates linear staining.

Microscopic polyangitis (C) is a small vessel vasculitis affecting the arterioles, venules and capillaries. This vasculitis is associated with focal necrotizing glomerulonephritis as well as the presence of perinuclear ANCA (p-ANCA) in the circulation directed towards cytoplasmic myeloperoxidase.

Lupus nephritis (D) is characterized by the deposition of IgG, IgM, IgA and C3 in the sub-endothelial segment of the glomerular basement membrane and in the mesangium.

Membranoproliferative glomerulonephritis (F) is defined by mesangial cell proliferation with thickening of the capillaries. Two types exist: type 1 in which there is classical and alternative complement pathway activation and type 2 that is associated with only alternative pathway activation.

Goodpasture’s syndrome (I) is characterized by the presence of anti-glomerular basement membrane proteins, specifically targeting type IV collagen. Pulmonary features include cough, dyspnoea and haemoptysis; renal features include haematuria, acute renal failure and nephrotic syndrome.

How well did you know this?
1
Not at all
2
3
4
5
Perfectly
32
Q
  1. Nephritic disorders
A Minimal change disease
B Wegener’s granulomatosis
C Microscopic polyangitis
D Lupus nephritis
E IgA nephropathy
F Membranoproliferative glomerulonephritis
G Rapidly progressive glomerulonephritis
H Post-streptococcal glomerulonephritis
I Goodpasture’s syndrome

3 A 25-year-old man presents to his GP with symptoms and signs of nephritic syndrome. The patient had a sore throat 2 weeks previously. Blood tests reveal anti-streptolysin titre is high, while IgA levels are normal.

A

3) H

Post-streptococcal glomerulonephritis (H) is usually caused by a preceding group A beta haemolytic streptococcus pharyngitis. Anti-streptolysin O titre (ASOT) will be raised. Pathological hallmarks of post-streptococcal glomerulonephritis include diffuse hypercellularity and diffuse swelling of the mesangium and glomerular capillaries. Influx of neutrophils and macrophages may reveal crescent formation on histology. Direct immunofluorescence reveals the sub-epithelial deposition of IgG and C3. The condition usually subsides with supportive treatment, including antibiotic therapy to combat the outstanding infection.

Microscopic polyangitis (C) is a small vessel vasculitis affecting the arterioles, venules and capillaries. This vasculitis is associated with focal necrotizing glomerulonephritis as well as the presence of perinuclear ANCA (p-ANCA) in the circulation directed towards cytoplasmic myeloperoxidase.

Lupus nephritis (D) is characterized by the deposition of IgG, IgM, IgA and C3 in the sub-endothelial segment of the glomerular basement membrane and in the mesangium.

Membranoproliferative glomerulonephritis (F) is defined by mesangial cell proliferation with thickening of the capillaries. Two types exist: type 1 in which there is classical and alternative complement pathway activation and type 2 that is associated with only alternative pathway activation.

Goodpasture’s syndrome (I) is characterized by the presence of anti-glomerular basement membrane proteins, specifically targeting type IV collagen. Pulmonary features include cough, dyspnoea and haemoptysis; renal features include haematuria, acute renal failure and nephrotic syndrome.

How well did you know this?
1
Not at all
2
3
4
5
Perfectly
33
Q
  1. Nephritic disorders
A Minimal change disease
B Wegener’s granulomatosis
C Microscopic polyangitis
D Lupus nephritis
E IgA nephropathy
F Membranoproliferative glomerulonephritis
G Rapidly progressive glomerulonephritis
H Post-streptococcal glomerulonephritis
I Goodpasture’s syndrome

2 A 24-year-old man presents to his GP with a few days’ history of blood in his urine. Urinary investigations reveal the presence of proteinuria, red and white cell casts and dysmorphic red cells. The patient’s notes state that he was diagnosed with pharyngitis in the previous week. Blood tests reveal a raised IgA level.

A

2) E

IgA nephropathy (Berger’s disease; E) is the most common cause of glomerunephritis in the developed world. The condition occurs after a gastrointestinal or upper respiratory infection; potential offenders are postulated to include Haemophilus influenzae, hepatitis B virus and cytomegalovirus. Antigenic targets for IgA are thought to include collagen, fibronectin and laminin. Characteristically there is mesangial proliferation with deposition of IgA together with alternative pathway factors C3 and properdin. Blood tests will reveal a raised IgA level. Henoch–Schonlein purpura has a similar pathogenesis to IgA nephropathy but presents in children and has extra-renal clinical features.

Microscopic polyangitis (C) is a small vessel vasculitis affecting the arterioles, venules and capillaries. This vasculitis is associated with focal necrotizing glomerulonephritis as well as the presence of perinuclear ANCA (p-ANCA) in the circulation directed towards cytoplasmic myeloperoxidase.

Lupus nephritis (D) is characterized by the deposition of IgG, IgM, IgA and C3 in the sub-endothelial segment of the glomerular basement membrane and in the mesangium.

Membranoproliferative glomerulonephritis (F) is defined by mesangial cell proliferation with thickening of the capillaries. Two types exist: type 1 in which there is classical and alternative complement pathway activation and type 2 that is associated with only alternative pathway activation.

Goodpasture’s syndrome (I) is characterized by the presence of anti-glomerular basement membrane proteins, specifically targeting type IV collagen. Pulmonary features include cough, dyspnoea and haemoptysis; renal features include haematuria, acute renal failure and nephrotic syndrome.

How well did you know this?
1
Not at all
2
3
4
5
Perfectly
34
Q
  1. Nephritic disorders
A Minimal change disease
B Wegener’s granulomatosis
C Microscopic polyangitis
D Lupus nephritis
E IgA nephropathy
F Membranoproliferative glomerulonephritis
G Rapidly progressive glomerulonephritis
H Post-streptococcal glomerulonephritis
I Goodpasture’s syndrome

1 A 50-year-old woman presents to accident and emergency with haematuria. Blood tests demonstrate deranged renal function and further tests reveal the presence of circulating cANCA antibodies. The patient is noted to have a saddle-shaped nose.

A

1) B

Wegener’s granulamatosis (B) is a systemic vasculitis characterized clinically by epistaxis, haemoptysis and haematuria. Wegener’s granulomatosis is defined by the presence of cytoplasmic anti-neutrophil cytoplasmic antibodies (cANCA). c-ANCA is directed towards proteinase 3 (PR3), an enzyme normally present within the cytoplasm of neutrophils. It is proposed that an infection is the trigger for the disease, which causes circulating neutrophils to become adherent to the endothelium and upregulation of PR3 on the cell surface. Vasculitis is mediated by both direct effect of PR3 on the endothelium as well as cANCA–PR3 immune complex deposition.

Microscopic polyangitis (C) is a small vessel vasculitis affecting the arterioles, venules and capillaries. This vasculitis is associated with focal necrotizing glomerulonephritis as well as the presence of perinuclear ANCA (p-ANCA) in the circulation directed towards cytoplasmic myeloperoxidase.

Lupus nephritis (D) is characterized by the deposition of IgG, IgM, IgA and C3 in the sub-endothelial segment of the glomerular basement membrane and in the mesangium.

Membranoproliferative glomerulonephritis (F) is defined by mesangial cell proliferation with thickening of the capillaries. Two types exist: type 1 in which there is classical and alternative complement pathway activation and type 2 that is associated with only alternative pathway activation.

Goodpasture’s syndrome (I) is characterized by the presence of anti-glomerular basement membrane proteins, specifically targeting type IV collagen. Pulmonary features include cough, dyspnoea and haemoptysis; renal features include haematuria, acute renal failure and nephrotic syndrome.

How well did you know this?
1
Not at all
2
3
4
5
Perfectly
35
Q
  1. Cell changes
    A 32-year-old man has a past medical history of severe gastro-oesophageal reflux disease. His most recent oesophageal biopsy shows a columnar epithelium with goblet cells suggestive of a diagnosis of Barrett’s oesophagus. What form of cell change is this also known as?
A Anaplasia
B Hyperplasia
C Metaplasia
D Dysplasia
E Neoplasia
A

C

The normal oesophagus is lined by stratified squamous epithelium and the squamo-columnar junction lies 2cm above the gastro-oesophageal junction and is recognized by an irregular white line known as the Z line. Barrett’s oesophagus occurs due to long standing reflux, and is the re-epithelialization by metaplastic columnar epithelium with goblet cells replacing normal squamous epithelium. This is known as metaplasia (C), which is the conversion from one type of differentiated tissue to another. It is reversible and often represents an adaptive response to environmental stress. Surveillance is crucial with repeated biopsy to detect a potential adenocarcinoma early, which is becoming more common than squamous cell carcinomas.

Metaplasia can lead to dysplasia (D), which is an abnormal pre-malignant disordered pattern of growth. The cells are altered in size, shape and organization.

If it leads to neoplasia (E), histologically there will be an abnormal proliferation and the process of tumour growth is detected.

Anaplasia (A) is defined as dedifferentiation or the reversion of differentiated cells and is characteristic of malignant neoplasms, implying loss of structure or function, which is not the case here.

Hyperplasia (B) is a physiological proliferation and increase in the number of cells in a tissue or organ, which is not consistent with Barrett’s oesophagus.

How well did you know this?
1
Not at all
2
3
4
5
Perfectly
36
Q
  1. Breast pathology
A Mastitis
B Phylloides tumour
C Fibroadenoma
D Duct ectasia
E Ductal carcinoma in situ
F Gynaecomastia
G Fibrocystic disease
H Fat necrosis
I Infiltrating ductal carcinoma

5 A 58-year-old woman presents to her GP with a painful lump in her right breast. On examination there is also evidence of peau d’orange.

A

5) I

Infiltrating ductal carcinoma (IDC; I) is an invasive cancer and therefore penetrates the basement membrane. IDC is also called no special type and usually results from DCIS. Macroscopically IDC has a scirrhous look whereby the centre is very fibrous giving a dense white appearance. IDC has the worst prognosis compared to all other invasive carcinomas (medullary, mucinous, tubular and papillary carcinoma). Features of invasive carcinoma also include peau d’orange, Paget’s disease of the breast, tethering, nipple retraction, lymphadenopathy, ulceration of the mass and pain.

Mastitis (A) occurs in breast-feeding mothers. As a result of cracked nipples, bacteria such as Staphylococcus aureus are able to enter. On examination, the area is tender to touch, erythematous and oedematous.

Gynaecomastia (F) is defined as enlargement of male breasts due to epithelial hyperplasia. Causes include: malnutrition, spironolactone, ketoconazole and cirrhosis.

Fibrocystic disease (G) is usually a peri-menopausal disease that regresses after menopause. It occurs due to irregular menstrual cycles that result in unbalanced oestrogen.

Fat necrosis (H) occurs after trauma to the breast resulting in a tender lump consisting of necrosed fat surrounded by macrophages.

How well did you know this?
1
Not at all
2
3
4
5
Perfectly
37
Q
  1. Breast pathology
A Mastitis
B Phylloides tumour
C Fibroadenoma
D Duct ectasia
E Ductal carcinoma in situ
F Gynaecomastia
G Fibrocystic disease
H Fat necrosis
I Infiltrating ductal carcinoma

4 A 60-year-old woman presents to her GP with a 5.5 cm mobile lump in her right breast. Biopsy reveals an ‘artichoke-like’ appearance.

A

4) B

Phylloides tumours (B) are similar to fibroadenomas as they are mixed; they arise from stroma and duct epithelium. They are also similar to fibroadenomas as they are discrete, well-circumscribed and mobile. They differ in that they are usually greater than 5cm, occur in women over the age of 40 years and can be malignant. Histological investigation reveals an ‘artichoke-like’ appearance as the stroma pushes up on the epithelium to form clubs.

Mastitis (A) occurs in breast-feeding mothers. As a result of cracked nipples, bacteria such as Staphylococcus aureus are able to enter. On examination, the area is tender to touch, erythematous and oedematous.

Gynaecomastia (F) is defined as enlargement of male breasts due to epithelial hyperplasia. Causes include: malnutrition, spironolactone, ketoconazole and cirrhosis.

Fibrocystic disease (G) is usually a peri-menopausal disease that regresses after menopause. It occurs due to irregular menstrual cycles that result in unbalanced oestrogen.

Fat necrosis (H) occurs after trauma to the breast resulting in a tender lump consisting of necrosed fat surrounded by macrophages.

How well did you know this?
1
Not at all
2
3
4
5
Perfectly
38
Q
  1. Breast pathology
A Mastitis
B Phylloides tumour
C Fibroadenoma
D Duct ectasia
E Ductal carcinoma in situ
F Gynaecomastia
G Fibrocystic disease
H Fat necrosis
I Infiltrating ductal carcinoma

3 A 54-year-old woman presents to her GP with a single lump in her left breast. A mammogram reveals a focal area of calcification.

A

3) E

Ductal carcinomas in situ (DCIS; E) occur in pre- or post-menopausal women. They are usually unilateral and unifocal. On mammogram, microcalcification may be visible secondary to central necrosis. Microscopic features include the presence of central necrosis and pleomorphic nuclei. In contrast, lobular carcinoma in situ (LCIS) usually occurs mainly in premenopausal women and is bilateral and multifocal. No calcification occurs and hence there is no detection on mammogram. Histologically there is no necrosis and uniform nuclei are present.

Mastitis (A) occurs in breast-feeding mothers. As a result of cracked nipples, bacteria such as Staphylococcus aureus are able to enter. On examination, the area is tender to touch, erythematous and oedematous.

Gynaecomastia (F) is defined as enlargement of male breasts due to epithelial hyperplasia. Causes include: malnutrition, spironolactone, ketoconazole and cirrhosis.

Fibrocystic disease (G) is usually a peri-menopausal disease that regresses after menopause. It occurs due to irregular menstrual cycles that result in unbalanced oestrogen.

Fat necrosis (H) occurs after trauma to the breast resulting in a tender lump consisting of necrosed fat surrounded by macrophages.

How well did you know this?
1
Not at all
2
3
4
5
Perfectly
39
Q
  1. Breast pathology
A Mastitis
B Phylloides tumour
C Fibroadenoma
D Duct ectasia
E Ductal carcinoma in situ
F Gynaecomastia
G Fibrocystic disease
H Fat necrosis
I Infiltrating ductal carcinoma

2 A 35-year-old woman presents to her GP with a soft 3 cm mobile mass in her left breast. The patient suggests the size of the lump fluctuates with her menstrual cycle.

A

2) C

Fibroadenoma (C) is the most common benign breast tumour. Fibroadenomas arise from stroma as well as lobules and hence are mixed tumours. They characteristically grow rapidly in pregnancy and during the menstrual cycle as lobules are oestrogen driven; conversely, fibroadenomas regress at menopause due to the lack of oestrogen. On examination, fibroadenomas are very mobile (sometimes known as a breast mouse), well circumscribed, discrete and usually less than 5cm. They tend to be soft in a young female and firm in elderly women (as stroma becomes more fibrous).

Mastitis (A) occurs in breast-feeding mothers. As a result of cracked nipples, bacteria such as Staphylococcus aureus are able to enter. On examination, the area is tender to touch, erythematous and oedematous.

Gynaecomastia (F) is defined as enlargement of male breasts due to epithelial hyperplasia. Causes include: malnutrition, spironolactone, ketoconazole and cirrhosis.

Fibrocystic disease (G) is usually a peri-menopausal disease that regresses after menopause. It occurs due to irregular menstrual cycles that result in unbalanced oestrogen.

Fat necrosis (H) occurs after trauma to the breast resulting in a tender lump consisting of necrosed fat surrounded by macrophages.

How well did you know this?
1
Not at all
2
3
4
5
Perfectly
40
Q
  1. Breast pathology
A Mastitis
B Phylloides tumour
C Fibroadenoma
D Duct ectasia
E Ductal carcinoma in situ
F Gynaecomastia
G Fibrocystic disease
H Fat necrosis
I Infiltrating ductal carcinoma

1 A 55-year-old parous woman presents to her GP with a 2-week history of green discharge from her right nipple.

A

1) D

Duct ectasia (D) is defined as the chronic ductal inflammation due to acini secretions that become clogged in the ducts causing them to dilate and rupture. This leads to a green/white discharge being produced. Duct ectasia occurs in women older than 40 who have had children. It is an important diagnosis to make as it mimics breast cancer; the presentation may be nipple retraction due to fibrosis and bloody discharge secondary to rupture of the ducts.

Mastitis (A) occurs in breast-feeding mothers. As a result of cracked nipples, bacteria such as Staphylococcus aureus are able to enter. On examination, the area is tender to touch, erythematous and oedematous.

Gynaecomastia (F) is defined as enlargement of male breasts due to epithelial hyperplasia. Causes include: malnutrition, spironolactone, ketoconazole and cirrhosis.

Fibrocystic disease (G) is usually a peri-menopausal disease that regresses after menopause. It occurs due to irregular menstrual cycles that result in unbalanced oestrogen.

Fat necrosis (H) occurs after trauma to the breast resulting in a tender lump consisting of necrosed fat surrounded by macrophages.

How well did you know this?
1
Not at all
2
3
4
5
Perfectly
41
Q
  1. Renal pathology
A Nephritic syndrome
B Wegener’s granulomatosis
C Membranous glomerulonephritis
D Acute tubular necrosis
E Minimal change glomerulo-nephritis
F Goodpasture’s syndrome
G IgA nephropathy
H Nephrotic syndrome
I Focal segmental glomerulo-nephritis

A 62-year-old woman on the Care of the Elderly ward is found to have new onset ankle swelling. A urine dipstick demonstrates proteinuria and the only blood abnormality is a low albumin level.

A

H

Nephrotic syndrome (H) is the combination of proteinuria, hypoalbuminaemia and oedema (with associated hyperlipidaemia and lipiduria). Primary causes such as IgA nephropathy are most common in children, whereas systemic causes such as diabetes and SLE are more common in adults. Damage to the glomerulus causes increased permeability and hence proteins pass into the tubules leading to proteinuria and hypo-albuminaemia. A reduced oncotic pressure therefore causes oedema. The liver compensates by producing increased amounts of lipids which are then excreted via the damaged kidneys causing lipiduria.

Wegener’s granulamatosis (B) is a c-ANCA mediated pauci-immune RPGN, creating an absent/scant pattern on IgG and C3 immunofluorescence staining.

Membranous glomerulonephritis (C) is defined on histological investigation as a thickened glomerular basement membrane, spike/dome protrusions, sub-epithelial immunoglobulin and a granular staining pattern.

Minimal change glomerulonephritis (E) appears normal with light microscopy but podocyte effacement is visible on electron microscopy. Most cases will respond to steroid treatment.

Focal segmental glomerulonephritis (I) appears with obliterated lumen and podocyte effacement on histological examination. Focal segmental glomerulonephritis is not responsive to steroids.

How well did you know this?
1
Not at all
2
3
4
5
Perfectly
42
Q
  1. Renal pathology
A Nephritic syndrome
B Wegener’s granulomatosis
C Membranous glomerulonephritis
D Acute tubular necrosis
E Minimal change glomerulo-nephritis
F Goodpasture’s syndrome
G IgA nephropathy
H Nephrotic syndrome
I Focal segmental glomerulo-nephritis

4 An 8-year-old girl presents to accident and emergency with frank haematuria. Her parents state that she had just recovered from a throat infection 2 days previously.

A

4) G

IgA nephropathy (Berger’s disease; G) usually occurs 1–4 days after a respiratory or gastrointestinal infection (mucosal defence is primarily IgA). IgA is deposited in the mesangium which may lead to RPGN. On immunofluorescence a granular staining of IgG and C3 will demonstrate IgA nephropathy (other conditions causing this pattern are SLE, Henoch–Schlönein purpura, post-streptococcal infection and Alport’s syndrome). There is frank haematuria in 50% of cases and microscopic haematuria in 50% of patients.

Wegener’s granulamatosis (B) is a c-ANCA mediated pauci-immune RPGN, creating an absent/scant pattern on IgG and C3 immunofluorescence staining.

Membranous glomerulonephritis (C) is defined on histological investigation as a thickened glomerular basement membrane, spike/dome protrusions, sub-epithelial immunoglobulin and a granular staining pattern.

Minimal change glomerulonephritis (E) appears normal with light microscopy but podocyte effacement is visible on electron microscopy. Most cases will respond to steroid treatment.

Focal segmental glomerulonephritis (I) appears with obliterated lumen and podocyte effacement on histological examination. Focal segmental glomerulonephritis is not responsive to steroids.

43
Q
  1. Renal pathology
A Nephritic syndrome
B Wegener’s granulomatosis
C Membranous glomerulonephritis
D Acute tubular necrosis
E Minimal change glomerulo-nephritis
F Goodpasture’s syndrome
G IgA nephropathy
H Nephrotic syndrome
I Focal segmental glomerulo-nephritis

3 A 64-year-old man on the Care of the Elderly ward is found to be in acute renal failure secondary to statin-related rhabdomyolysis. Urinalysis reveals the presence of ‘muddy’ casts.

A

3) D

Acute tubular necrosis (D) is defined as damage to the tubular epithelium leading to acute renal failure. Ischaemic or toxic injury reduces GFR in three ways:

1) Loss of polarity (loss of membrane channels reduces sodium reabsorption; more sodium reaches macula densa constricting afferent arteriole, hence reducing GFR);
2) Glomerular back-pressure (formation of casts in distal convoluted tubule creates back- pressure reducing GFR); and
3) Interstitial leakage (back-pressure forces fluid into interstitium causing swelling and compression of tubules).

Wegener’s granulamatosis (B) is a c-ANCA mediated pauci-immune RPGN, creating an absent/scant pattern on IgG and C3 immunofluorescence staining.

Membranous glomerulonephritis (C) is defined on histological investigation as a thickened glomerular basement membrane, spike/dome protrusions, sub-epithelial immunoglobulin and a granular staining pattern.

Minimal change glomerulonephritis (E) appears normal with light microscopy but podocyte effacement is visible on electron microscopy. Most cases will respond to steroid treatment.

Focal segmental glomerulonephritis (I) appears with obliterated lumen and podocyte effacement on histological examination. Focal segmental glomerulonephritis is not responsive to steroids.

44
Q
  1. Renal pathology
A Nephritic syndrome
B Wegener’s granulomatosis
C Membranous glomerulonephritis
D Acute tubular necrosis
E Minimal change glomerulo-nephritis
F Goodpasture’s syndrome
G IgA nephropathy
H Nephrotic syndrome
I Focal segmental glomerulo-nephritis

2 A 42-year-old man presents to accident and emergency with an episode of haemoptysis and haematuria. Blood tests reveal he is in acute renal failure. Once the patient is stable a renal biopsy demonstrates a crescent morphology on immunofluorescence.

A

2) F

Goodpasture’s syndrome (F) is an anti-glomerular basement membrane disease that causes rapidly progressive glomerulonephritis (RPGN). RPGN all demonstrate a crescent sign on biopsy, which represents the proliferation of macrophages and parietal cells in the Bowman’s space. There are numerous causes of RPGN; these can be differentiated by looking at the IgG and C3 deposition pattern on immunofluorescence. Goodpasture’s syndrome occurs due to IgG against the A3 chain of type-4 collagen, creating a linear pattern on immunofluorescence.

Wegener’s granulamatosis (B) is a c-ANCA mediated pauci-immune RPGN, creating an absent/scant pattern on IgG and C3 immunofluorescence staining.

Membranous glomerulonephritis (C) is defined on histological investigation as a thickened glomerular basement membrane, spike/dome protrusions, sub-epithelial immunoglobulin and a granular staining pattern.

Minimal change glomerulonephritis (E) appears normal with light microscopy but podocyte effacement is visible on electron microscopy. Most cases will respond to steroid treatment.

Focal segmental glomerulonephritis (I) appears with obliterated lumen and podocyte effacement on histological examination. Focal segmental glomerulonephritis is not responsive to steroids.

45
Q
  1. Coeliac disease
    A 26-year-old woman presents with fatigue ‘all the time’. She has a family history of coeliac disease and blood tests reveal hypochromic, microcytic anaemia. She is referred to the gastroenterology clinic for tests. The gold standard investigation is the duodenal biopsy, which is carried out after positive serological testing. Which current serological testing and histopathology findings in the options below are most consistent with a coeliac disease diagnosis?

A Anti-reticulin antibodies only/villous atrophy, crypt hyperplasia, increased intraepithelial lymphocytes
B Anti-gliadin antibodies only/no villous atrophy, crypt hyperplasia, decreased intraepithelial lymphocytes
C Anti-endomysial antibodies only/villous atrophy, crypt hyperplasia, increased intraepithelial lymphocytes
D Anti-endomysial antibodies and anti-tissue transglutaminase antibodies/ villous atrophy, crypt hyperplasia, increased intraepithelial lymphocytes
E Anti-endomysial antibodies and anti-tissue transglutaminase antibodies/villous atrophy, no evidence of crypt hyperplasia, increased intraepithelial lymphocytes

A

D
The following investigations are key in diagnosing coeliac disease: inflammatory markers (CRP and ESR), current serological tests (anti-endomysial antibodies and anti-tissue transglutaminase antibodies), and upper GI endoscopy and distal duodenal biopsies.

There are 4 serological investigations for coeliac disease; 2 are out-dated because of non-specificity and unreliability (anti-reticulin antibodies and anti-gliadin antibodies) and 2 are currently used (anti-endomysial antibodies and anti-tissue transglutaminase antibodies). The IgA autoantibody to endomysial cells is both specific and sensitive, however interpretation requires considerable operator expertise. The most sensitive (at 90–94% and specific (~95%) for coeliac disease is said to be the IgA anti-tissue transglutaminase (anti-TTG) antibody.

The endoscopy, if positive, will show mucosal folds and lack of villi. Histopathology will show villous atrophy, crypt hyperplasia, and increased intraepithelial lymphocytes. Normally, enterocytes are constantly shed from the tips of the villi and replenished by migration of cells up the villi from the proliferative compartment in the crypts. With higher rates of cell loss, a stage is soon reached when the increased proliferative compartment (evidenced by elongation, hyper-cellularity and high mitotic activity of the crypts) cannot maintain a normal number of maturing and functioning ‘end cells’, the size of this compartment diminishes and villous atrophy results. This degenerate epithelial surface is infiltrated by large numbers of T lymphocytes, which are further enhanced by the local production of TNF-alpha.

46
Q
  1. Non-neoplastic bone tumours
    A 27-year-old woman has developed pain in her right proximal femur. She has a history of intermittent hip pain since childhood. An X-ray has demonstrated a ‘soap bubble’ appearance indicative of osteolysis and a characteristic shepherd’s crook deformity. The biopsy would show irregular trabeculae of woven bone said to resemble Chinese letters. What is the most likely diagnosis?
A Non-ossifying fibroma
B Fibrous dysplasia
C Giant cell reparative granuloma
D Ossifying fibroma
E Simple bone cyst
A

B

Fibrous dysplasia (B) is a benign disorder of children and young adults, whereby lesions composed of fibrous and bony tissue develop usually in the ribs, femur, tibia or skull. It mostly presents as bone pain and weakness in female patients under 30 years of age and results from congenital dysplasia of bone, consistent with the patient’s history. 
There are three forms: the more common monostotic form (lesions localized to only one bone), polyostotic (multiple lesions) and McCune–Albright syndrome, which also has endocrine manifestations. Shepherd’s crook deformity refers to a varus angulation of the proximal femur commonly seen in the femoral involvement of polyostotic fibrous dysplasia. Histologically, it is characterized by loose fibrous tissue with metaplastic immature or woven bone trabeculae arranged in a ‘Chinese letters’ formation.

Non-ossifying fibroma (A) is often asymptomatic and merely an incidental finding on X-ray and occurs in an even younger group of patients.

Ossifying fibroma (D) is a benign, fibrous tumour with reactive bone formation that shows local aggressive behaviour. Lesions are found in the mandible for adults, and the tibia for children.

Giant-cell reparative granuloma (C) is an uncommon benign reactive intraosseous lesion and can occur in the skull, jaw, hand and foot. The histology resembles giant cell tumour of bone (see below).

A simple bone cyst (E) is common in the proximal metaphysis of the humerus and asymptomatic unless there has been a fracture.

47
Q
  1. Benign bone tumours
    A 36-year-old man presents with swelling of his middle finger and subsequently a fracture. His X-ray shows cotton wool calcification and histopathology shows evidence of a tumour composed of benign hyaline cartilage. It is believed that he has only a very slight risk of malignant transformation. What is the most likely diagnosis?
A Osteochondroma
B Multiple myeloma
C Osteoid osteoma
D Giant cell tumour
E Enchondroma
A
E
Enchondroma (E) is a benign intramedullary cartilage tumour usually found in the central mature hyaline cartilage of the short tubular bones of the hands and feet. It may present at any age (average is 40 years) and is often asymptomatic, but some patients present with pain, fracture or swelling of the affected area. There are lytic lesions on X-rays that usually contain variably calcified chondriod matrix and histopathologi- cal findings showing bluish-grey lobules of hyaline cartilege. There may be a thin layer of lamellar bone surrounding the cartilage nodules but no permeation of pre-existing host bone which is a positive sign that the lesion is benign. Each potential enchondroma needs to be evaluated on imaging and histology to distinguish it from low-grade chondrosarcoma.

Osteochondroma (A) is the most common skeletal neoplasm, often affect- ing boys in their teens and often found in the long bones. When the growth plates close in late adolescence, there is usually no further growth of the osteochondroma. There is a low risk of malignancy in osteochondromas associated with syndromes that involve multiple lesions such as Ollier’s disease or Maffucci’s syndrome.

Osteoid osteoma (C) is a small, benign lesion surrounded by a zone of reactive bone. It mostly occurs in the long bones, with a very distinct clinical picture in that pain is dull, worse at night and relieved by aspirin. Similar radiology and histology are seen in osteoblastomatomours that are >1 cm diameter and that commonly affect the vertebrae.

Pathological fracture is a common presentation of giant cell tumour (D), which is more common in females, is locally aggressive and can metastasize. It does not occur in the imma-ture skeleton. It is more common in the metaphysics of the long bones, especially the distal femur and proximal tibia. There is no calcification on X-ray and, histologically, there would be numerous multinucleated giant cells. The mono-nuclear cell population is the neoplastic cells.

Multiple myeloma (B) is highly unlikely as it is a different classification of bone tumours altogether and is considered as a blood cell malignancy rather than a primary bone tumour. It is a malignant tumour of plasma cells that causes widespread osteolytic bone damage. It may be in are bone only when it is known as plasmacytoma rather than myeloma

48
Q
  1. Malignant bone tumours
    An 18-year-old man presents with pain and a mass in his right knee. His X-ray shows an ill defined mass in the metaphyseal region of the distal femur that is sclerotic and lytic. There is also an elevated periosteum (known as a Codman’s triangle). Prognosis is said to be poor and the treatment required is multi-disciplinary involving intensive chemotherapy and surgery. In cytology, these tumour cells will be positive for alkaline phosphatase. What is the most likely diagnosis?
A Osteosarcoma
B Chondrosarcoma
C Fibrosarcoma
D Malignant fibrous histiocytoma
E Ewing’s sarcoma
A

A

Osteosarcoma (A) is the most common primary bone malignancy that is aggressive with poor prognosis (60 per cent of patients with 5-year survival). The vast majority of these tumours occur in teenagers and young adults. The most common presentation is pain and a mass, which occurs near a joint such as the knee, with the distal femur and proximal tibia being the most common sites. X-rays show a mixed sclerotic and lytic lesion in the metaphysis that may permeate the bone causing a soft tissue mass and a periosteal reaction. Bone formation within the tumour is characteristic of osteosarcoma and is usually visible on X-rays. Bone alkaline phosphatase has significant value in diagnosing osteosarcoma, also indicating chemotherapy effectiveness and prognosis.

Chondrosarcomas (B) usually affect older patients. The prognosis is vari- able depending on the grade of tumour. They usually grow slowly and not only affect long bones but also the ribs, spine and pelvis. The only treatment is surgical excision as both chemotherapy and radiotherapy are ineffective.

Fibrosarcomas (C) and malignant fibrous histiocytomas (D) are both spindle cell malignant tumours, arising from stromal cells and collectively they make up the majority of soft tissue sarcomas.

Ewing’s sarcoma (E) is one of the family of malignant small round cell tumours. It affects children and teenagers and has a characteristic chromosomal translocation (t11;22)(q24;q12). It is negative for alkaline phosphatase and positive for CD99 (MIC2) immunostain. It is more commonly found in the pelvis and diaphysis/metaphysis of long bones than around the knee. Widespread metastases and bone marrow involvement are frequent.

49
Q
  1. NHS cervical screening programme
    A 25-year-old woman is due for her cervical smear test. Which method of cyto-pathology is going to be used?
A FNA
B Ultrasound guided FNA
C Washings
D Brushings
E Liquid based cytology
A

E

Liquid based cytology (E) is an important method of preparing cervical samples for examination under the microscope. The sample is collected in the standard brushings method using a spatula but rather than smearing the sample onto a microscope slide as before, the head of the spatula is broken off into a small glass vial containing preservative fluid. This is then sent to the laboratory and obscuring material, e.g. mucus and pus, is removed and a representative sample of cells is deposited onto a slide for examination. The vial can also be tested for the human papillomavirus and other sexually transmitted diseases. Fine needle aspiration with or without ultrasound (A, B) is only suitable for lesions that are easily palpable by hand or easily detected via imaging; common sites include breast, thyroid and lymph nodes. They are not suitable for cervical samples.

Washings (C) and brushings (D) are types of exfoliative cytology. This is used when cells are dislodged or spontaneously shed such as bronchial washings. The use of the spatula in the smear test is a form of brushing but liquid based cytology involves a crucial additional component that brings many benefits to patients and staff, including quicker results and fewer inadequate tests.

50
Q
  1. Obstructive lung disease
    A 57-year-old man who is a heavy smoker presents to his GP with gradually worsening dyspnoea and cough productive of green sputum. On examination, he is cyanosed, tachypnoeic and wheezing. What is the most likely diagnosis?
A Chronic bronchitis
B Pulmonary embolus
C Asthma
D Bronchiolitis
E Emphysema
A

A

Obstructive lung disease is characterized by airway obstruction and a decreased FEV1/FVC ratio. Chronic obstructive pulmonary disease (COPD) includes chronic bronchitis (A) and emphysema (E). While the two often coexist, a predominantly chronic bronchitis cough produces copious amounts of sputum, and infection is consistent with this patient, while sputum and infection are only occasional in emphysema. Chronic bronchitis is the damage caused to the airways/bronchi and the clinical definition is productive cough for at least 3 months per year for 2 consecutive years. Histopathology shows hypertrophy of mucous glands and goblet cell hyperplasia. Emphysema is alveolar parenchymal damage by activation of proteases (elastase) that are in turn activated by neutrophil/macrophage action secondary to cigarette smoking. Emphysema patients only have dyspnoea and no associated cough with no mucus excess. COPD is characteristically poorly reversible and gets progressively worse over time, consistent with this patient’s presentation.

In contrast, asthma (C) causes episodic wheezing, cough and dyspnoea. There is excessive mucus secretion in asthma, which poses a risk of mucus plug formation that can be fatal. Charcot–Leyden crystals indicative of eosinophilic inflammation would be found in these patients as the condition is under the allergic/immunological diseases umbrella.

Pulmonary embolus (B) is not an obstructive lung disorder but vascular. Its presentation is acute with pleuritic chest pain and progressive dyspnoea.

Bronchiolitis (D) is a paediatric presentation and the inflammation is often caused by a virus, e.g. respiratory syncytial virus.

51
Q
  1. Pneumonia
    A 57-year-old man presents to accident and emergency with dyspnoea, fever, cough and purulent sputum. Histopathology confirms widespread fibrinosuppurative consolidation on the left lower lobe and the top differential diagnosis is lobar pneumonia. Which organism is the most likely cause?
A Streptococcus pneumoniae
B Staphylococcus aureus
C Haemophilus influenzae
D Streptococcus pyogenes
E Mycobacterium tuberculosis
A

A

Lobar pneumonia is one of two anatomical classifications of pneumonia, the other being bronchopneumonia. In up to 95% of cases, the causative organism for lobar pneumonia is Streptococcus pneumoniae (A) type 1, 2, 3, and 7. The stages of pneumonia start with congestion in the first 24 hours; lobes are heavy, red and boggy due to hyperaemia, intra-alveolar fluid with scattered neutrophils and many bacteria. The second stage is red hepatization, with the lobe’s transformation to a liver-like mass, characteristic of lobar pneumonia. The red hepatization is blood-stained pulmonary exudate with many intra-alveolar neutrophils. Grey hepatization is when the lung is dry and firm because the red cells disintegrate and fibrinosuppurative exudates persist within the alveoli. Finally, the resolution stage involves the enzymatic digestion of the exudates in the alveoli and normal architecture remerges.

Pneumococcial infection can also cause bronchopneumonia, along with other low virulence organisms: Staphylococcus aureus (B), Haemophilus influenzae (C), Streptococcus pyogenes (D). Bronchopneumonia is more likely in patients with a compromised host defence system (e.g. elderly) and the pathology demonstrates a patchy bronchial and peribronchial distribution in the lower lobes.

Mycobacterium tuberculosis (E) is the cause of tuberculosis, which is an infectious granulomatous disease. A granuloma is a collection of histiocytes and macrophages with or without multinucleate giant cells.

52
Q
  1. Acute myocardial infarction
    A 57-year-old overweight patient suffers an acute myocardial infarction and subsequently dies. A post-morterm examination of the infarcted area shows extensive cell infiltration including polymorphs and macrophages. There is also extensive debris post necrosis and the cytoplasm is homogeneous making it difficult to see the outlines of the myocardial fibres. There is no evidence of collagenization or a scar. How long after the initial attack did the patient die?
A At the time of the attack (0–6 hours)
B Hours after the attack (6–24 hours)
C Days after the attack (1–4 days)
D Within the first 2 weeks of the attack (4–14 days)
E Weeks and months after (14 days +)
A

C

In the first 6 hours (A) in the evolution of an acute myocardial infarction (MI), the histology is normal. Necrotic cell death takes place between 6 and 24 hours (B). Pathologically, there is contraction band necrosis (dark red/pink wavy lines extending across the myocardial fibres), loss of nuclei in the myocardial cells and the start of a homogeneous appearing cytoplasm. At 1–4 days (C) following an acute MI, the start of an extensive acute inflammatory response takes place with cell infiltration. Debris is left by the necrosis in the previous stage and the cytoplasm is homo- geneous so that it is difficult to see the outlines of the myocardial fibres. Infiltration of polymorphs, and later macrophages, takes place. Removal of the debris takes place at about 5–10 days. At approximately 2 weeks (D), the area undergoes repair and a classic young scar with new capillaries is seen with early collagenization. Macrophages and myofibroblasts are present in large numbers. Some granulation tissue and collagen synthesis will continue on to the next stage. In the months (E) following an acute MI, the area starts to strengthen with the formation of a decellularizing scar. This established scar will be seen as a pale white collagenized area within the interstitium between myocardial fibres.

53
Q
  1. Connective tissue pathology
A Systemic lupus erythematosus
B Sjögren’s syndrome
C Diffuse scleroderma
D Amyloidosis
E Takayasu arteritis
F Dermatomyositis
G CREST syndrome
H Polymyositis
I Microscopic polyangitis

1 A 35-year-old woman is referred to the rheumatology clinic due to recent onset dysphagia. The patient also reports that her fingers have turned very pale and cold. One examination she is found to have tightening of the skin near her finger tips and small dilated vessels on her skin.

A

1) G

CREST syndrome (G), also known as limited scleroderma, represents a combination of conditions: calcinosis (calcium deposition in the skin), Raynaud’s disease (vasospasm of blood vessels in response to triggers such as cold), oesophageal dysmotility, sclerodactyly (thickening and tightening of skin surrounding fingers/hands) and telangiectasia (dilation of blood capillaries causing red marks on the surface of the skin). The pathogenesis relates to excessive release of PDGF causing widespread fibroblast activation and multi-organ fibrosis. Chronic fibrosis leads to initimal thickening of the microvasculature known as ‘onion skinning.’

Sjögren’s syndrome (B) is defined by the immune mediated destruction of the lacrimal and salivary glands that causes dry eyes and dry mouth.

In diffuse scleroderma (C), in contrast to limited scleroderma (CREST), there are extensive skin lesions, with an early visceral involvement and rapidly progressive course.

Takayasu arteritis (E) results in a thickened section of the aorta (usually the aortic arch). Features include no pulses and low blood pressure in the arms and cold hands.

Polymyositis (H) is part of the inflammatory myopathy spectrum. Similar to dermatomyositis there is proximal muscle weakness as well as heart and lung involvement, however with no dermatological lesions.

54
Q
  1. Connective tissue pathology
A Systemic lupus erythematosus
B Sjögren’s syndrome
C Diffuse scleroderma
D Amyloidosis
E Takayasu arteritis
F Dermatomyositis
G CREST syndrome
H Polymyositis
I Microscopic polyangitis

2 A 35-year-old woman with a history of recurrent miscarriages presents to her GP with joint pains. Blood tests reveal she is anti-double stranded DNA antibody positive.

A

2) A

Systemic lupus erythematosus (SLE; A) is a multi-system connective tissue disease that is antinuclear antibody (ANA) positive. The underlying pathology of SLE relates to failure in the regulatory mechanisms of self-tolerance. Autoantibodies form against nuclear components such as DNA, RNA and histones. This leads to complement activation and complex formation, which are deposited in organs. Cytology of tissues reveals haematoxylin bodies which are denatured nuclei that are produced when ANA bind to exposed nuclei. LE cells are also visible on microscopy; these are macrophages that have phagocytosed a haematoxylin body.

Sjögren’s syndrome (B) is defined by the immune mediated destruction of the lacrimal and salivary glands that causes dry eyes and dry mouth.

In diffuse scleroderma (C), in contrast to limited scleroderma (CREST), there are extensive skin lesions, with an early visceral involvement and rapidly progressive course.

Takayasu arteritis (E) results in a thickened section of the aorta (usually the aortic arch). Features include no pulses and low blood pressure in the arms and cold hands.

Polymyositis (H) is part of the inflammatory myopathy spectrum. Similar to dermatomyositis there is proximal muscle weakness as well as heart and lung involvement, however with no dermatological lesions.

55
Q
  1. Connective tissue pathology
A Systemic lupus erythematosus
B Sjögren’s syndrome
C Diffuse scleroderma
D Amyloidosis
E Takayasu arteritis
F Dermatomyositis
G CREST syndrome
H Polymyositis
I Microscopic polyangitis

3 A 68-year-old man presents to accident and emergency with symptoms suggestive of heart failure. All initial investigations do not determine an underlying cause. However, a tongue biopsy sample gains an apple-green birefringence under polarized light using Congo red stain.

A

3) D

Amyloidosis (D) occurs due to the extracellular deposition of fibrillar proteins that accumulate in tissues and organs. Amyloid proteins arise due to dysfunctional folding resulting in non-branching fibrils. Proteins aggregate into insoluble crossed beta-pleated sheet tertiary conformation. Amyloid proteins contain P-component which causes biopsy samples to characteristically gain an apple-green birefringence using polarized light and Congo red stain. Four major amyloid proteins exist: AA, derived from serum amyloid assisted protein and associated with inflammation; AL, derived from IgG light chains and associated with myeloma; AlphaBeta2, linked with Alzheimer’s disease; Beta2 microglobulin, associated with patients undergoing dialysis treatment.

Sjögren’s syndrome (B) is defined by the immune mediated destruction of the lacrimal and salivary glands that causes dry eyes and dry mouth.

In diffuse scleroderma (C), in contrast to limited scleroderma (CREST), there are extensive skin lesions, with an early visceral involvement and rapidly progressive course.

Takayasu arteritis (E) results in a thickened section of the aorta (usually the aortic arch). Features include no pulses and low blood pressure in the arms and cold hands.

Polymyositis (H) is part of the inflammatory myopathy spectrum. Similar to dermatomyositis there is proximal muscle weakness as well as heart and lung involvement, however with no dermatological lesions.

56
Q
  1. Connective tissue pathology
A Systemic lupus erythematosus
B Sjögren’s syndrome
C Diffuse scleroderma
D Amyloidosis
E Takayasu arteritis
F Dermatomyositis
G CREST syndrome
H Polymyositis
I Microscopic polyangitis

4 A 45-year-old woman presents to accident and emergency with signs suggestive of renal failure. She is found to be p-ANCA positive.

A

4) I

Microscopic polyangitis (I) is a small vessel vasculitis affecting the arterioles, venules and capillaries. The pathology involves a trigger factor such as microorganisms and drugs causing immune complex formation in a previously sensitized host. These immune complexes deposit in small vessels leading to neutrophil-related inflammation. Microscopic polyangitis affects the skin, heart, brain and kidneys. Histopathological features of affected vessels include fibrinoid necrosis that leads to fragmented neutrophilic nuclei within vessel walls. Microscopic polyangitis is associated with p-ANCA.

Sjögren’s syndrome (B) is defined by the immune mediated destruction of the lacrimal and salivary glands that causes dry eyes and dry mouth.

In diffuse scleroderma (C), in contrast to limited scleroderma (CREST), there are extensive skin lesions, with an early visceral involvement and rapidly progressive course.

Takayasu arteritis (E) results in a thickened section of the aorta (usually the aortic arch). Features include no pulses and low blood pressure in the arms and cold hands.

Polymyositis (H) is part of the inflammatory myopathy spectrum. Similar to dermatomyositis there is proximal muscle weakness as well as heart and lung involvement, however with no dermatological lesions.

57
Q
  1. Connective tissue pathology
A Systemic lupus erythematosus
B Sjögren’s syndrome
C Diffuse scleroderma
D Amyloidosis
E Takayasu arteritis
F Dermatomyositis
G CREST syndrome
H Polymyositis
I Microscopic polyangitis

5 A 52-year-old man presents to his GP with limb weakness and shortness of breath. A distinctive rash is noted around both eyes as well as plaques on the joints of his hands.

A

) F

Dermatomyositis (F) is an inflammatory myopathy that involves skeletal and thoracic muscles. Skeletal muscle involvement will lead to proximal muscle fatigue, especially in the hips and shoulders. Thoracic muscle involvement can affect the lungs (dyspnoea), heart (arrhythmia) and oesophagus (dysmotility). There is, however, sparing of the ocular muscles which differentiates dermatomyositis from myasthenia gravis. Dermatomyositis is also defined by a heliotrope rash (violet erythema around the periorbital region) and Gottron papules (violet scaly plaques over hand joints). Muscle inflammation will also cause an increased blood creatine kinase level.

Sjögren’s syndrome (B) is defined by the immune mediated destruction of the lacrimal and salivary glands that causes dry eyes and dry mouth.

In diffuse scleroderma (C), in contrast to limited scleroderma (CREST), there are extensive skin lesions, with an early visceral involvement and rapidly progressive course.

Takayasu arteritis (E) results in a thickened section of the aorta (usually the aortic arch). Features include no pulses and low blood pressure in the arms and cold hands.

Polymyositis (H) is part of the inflammatory myopathy spectrum. Similar to dermatomyositis there is proximal muscle weakness as well as heart and lung involvement, however with no dermatological lesions.

58
Q
  1. Diffuse alveolar damage
    A 27-year-old man with severe second degree burns is admitted to the ITU and develops severe shortness of breath and tachypnoea the next day. Diffuse alveolar damage is indicated in the histopathology report. What is the most likely diagnosis?
A Pulmonary oedema
B Acute respiratory distress syndrome
C Cryptogenic fibrosing alveolitis
D Bronchiectasis
E Chronic bronchitis
A

B
Acute respiratory distress syndrome (ARDS) (B) is a severe lung disease often developing with 24–48 hours post trauma, burns or sepsis. It is characterized by inflammation in the lung parenchyma impairing gas exchange and is therefore responsible for this patient’s symptoms. A less severe form is acute lung injury. Diffuse alveolar damage (DAD) is most commonly associated with ARDS.

Pulmonary oedema (A) is fluid accumulation within the alveolar spaces and interstitium but no damage is caused. These patients also present with shortness of breath.

Crytogenic fibrosing alveolitis (C) is a form of fibrosing lung disease that is chronic and progressive. It presents with shortness of breath and cough and is more likely to be seen in males over the age of 50. On histopathology, fibrosis and cyst formation are seen.

Bronchiectasis (D) is the permanent abnormal dilatation of bronchi and is very common in children in a post-infectious episode. The complications associated with it include recurrent infections, haemoptysis and pulmonary hypertension.

Chronic bronchitis (E) is defined as a chronic cough productive of sputum, for at least 3 months during a 2-year period. On histopathology, there is dilatation of airways and goblet cell hyperplasia.

59
Q
  1. Respiratory disease (1)
    A 47-year-old construction worker presents with a 6-month history of cough, haemoptysis and 5kg weight loss. He is a heavy smoker and a centrally located lesion is found on his chest X-ray. Histology showed keratinization and intercellular ‘prickles’. What is the most likely diagnosis?
A Tuberculosis
B Squamous cell carcinoma
C Mesothelioma
D Emphysema
E Large cell carcinoma
A

B

Squamous cell carcinoma (B) is one type of lung cancer within the non-small cell lung carcinoma (NSCLC) group. This type of cancer is most closely associated with a history of tobacco smoking and is more common in men. These tumours, if well-differentiated, show keratin pearls and cell junctions (or desmosomes), which form the characteristic intercellular ‘prickles’.

TB (A) is a granulomatous disease, whereby the histopathology shows a granuloma consisting of a collection of histiocytes/macrophages and/or multinucleate giant cells.

Mesothelioma (C) is a malignant tumour of the pleura and is associated with asbestos exposure decades earlier.

Emphysema (D) is unlikely in this case as it does not cause haemoptysis and weight loss. Histopathology shows perma- nent loss of the alveolar parenchyma distal to the terminal bronchiole and two key causes are cigarette smoking and alpha-1 antitrypsin deficiency.

Large cell carcinoma (E) is another type of NSCLC that is undifferentiated and diagnosis is made by exclusion of other tumour cells as there is no characteristic histological evidence.

60
Q
  1. Respiratory disease (2)
    A 55-year-old non-smoking woman presents to her GP with a 6-month history of cough, haemoptysis and 5kg weight loss. A chest X-ray showed the lesion is in the periphery and histopathology showed evidence of glandular differentiation and cytology showed mucin vacuoles. Mode of treatment most suitable is surgical. What is the most likely diagnosis?
A Small cell carcinoma
B Adenocarcinoma
C Large cell carcinoma
D Sarcoidosis
E Pneuomoconiosis
A

B

Adenocarcinoma (B) is the most common type of lung cancer in non- smokers and is usually seen in the periphery in the lungs, as opposed to small cell lung carcinoma (A) and squamous cell lung cancer, which both tend to be more centrally located. Adenocarcinomas often metastasize and are not as responsive to chemotherapy as small cell lung carcinomas.
The precursor of adenocarcinomas is termed atypical adenomatous hyperplasia (AAH) and this involves the proliferation of atypical cells lining the alveolar walls. Later they increase in size and become invasive. Small cell carcinoma has a very close association with smoking and paraneoplastic syndromes and is very chemo-sensitive. They are poorly differentiated and the mutations commonly involved are p53 and RB1.

Large cell carcinoma (C) is unlikely in this patient as histopathology would not show any characteristics or evidence of glandular differentiation.

Sarcoidosis (D) is a granulomatous multi-organ disease and surgery is not the mode of treatment.

Pneumoconiosis (E) is a type of fibrosing lung disease or ‘dusty lung’ caused by inorganic dust inhalation. It commonly affects the upper lobes, while asbestosis affects the lower lobes more severely. It is an alteration of the lung structure rather than neoplastic differentiation of cells and is therefore unlikely to be the diagnosis for this patient.

61
Q
  1. Erythema multiforme
    A 27-year-old man presents with fever, fatigue and a rash. He has also noted a few painful ulcers in his mouth. The rash is described as numerous round lesions about an inch in diameter on the face, trunk, arms and legs, diagnosed as erythema multiforme. What is the most likely diagnosis for this patient?
A Systemic lupus erythematosus
B Stevens–Johnson syndrome
C Pemphigoid
D Pityriasis rosea
E Contact dermatitis
A

B
Erythema multiforme is a pleomorphic skin eruption, with macules, papules, urticarial weals, vesicles and bullae. Its severest life-threatening form is related to Stevens–Johnson syndrome (B), believed to be mediated by the deposition of immune complexes in the microvasculature of the skin and oral mucous membranes following certain drugs, foods or infections.

Systemic lupus erythematosus (A) is much more common in females (ratio of 9:1) and patients develop the characteristic butterfly rash on the face. The systemic form affects the skin and internal organs, but discoid lupus erythematosus is a skin condition that presents with red, inflamed patches with scaling and a crusty appearance on the face, ears and scalp.

Pemphigoid (C) tends to present in the elderly with large tense bullae on flexor aspects of the forearms, groin and axillae, and histopathology reveals numerous eosinophils. IgG binds to basement membranes in pemphigoid (subepidermal bullae) but to desmosomes in pemphigus (intra-epidermal bullae).

Pityriasis rosea (D) is a salmon-pink scaly eruption on the trunk with the characteristic herald patch (a larger start lesion) and may be associated with a viral infection.

Contact dermatitis (E) is a localized skin reaction resulting from exposure and contact with an allergen/irritant, which this patient’s presentation lacks. The rash varies in severity from a red rash to blisters/weals to itchy burning skins.

62
Q
  1. Skin tumours
    A 55-year-old Australian man presents with a flat black lesion on his back that appears asymmetrical with an irregular border and 6mm in diameter. Breslow’s depth is 0.4mm. What is the most likely diagnosis?
A Malignant melanoma
B Basal cell carcinoma
C Squamous cell carcinoma
D Keratoacanthoma
E Bowen’s disease
A
A
Malignant melanoma (A) is the most life-threatening and aggres- sive form of skin tumour and must be monitored closely and treated assertively. It is key to examine for the ‘ABCDE’: asymmetry, border irregularity, colour, diameter and environment. The BRESLOW scale is used to quantify the tumour thickness and determine prognosis. 

Basal cell carcinoma (B) often presents as a ‘rodent ulcer’ on the face that is locally destructive and is described to have a pearly edge with telangiectasia and central ulceration.

Bowen’s disease (E) is a sunlight-induced skin disease that is considered to be a predisposing factor for squamous cell carcinoma (C).

Squamous cell carcinoma (SCC) of the skin begins as a small nodule that may become ulcerated and necrotic. Bleeding is quite common and the clinical presentation is highly variable.

Keratoacanthoma (D) is a benign lesion that is rapidly growing and dome-shaped. It arises from the pilosebaceous glands and often resembles SCC. However, the growth of a keratoacanthoma retains its smooth surface, unlike an SCC and a malignant melanoma. Bowen’s disease presents as an enlarging well-demarcated erythematous plaque with an irregular border but non-elevated. It is essentially an SCC in situ.

63
Q
  1. Inflammatory conditions
    A 23-year-old Irish man presents with an itchy blistering eruption on his buttocks and elbows. He also has diarrhoea and abdominal pain. Histopathology reveals papillary microabscesses and a neutrophilic infiltrate. He has a family history of gluten sensitivity. Which rash is most often associated with his presentation?
A Psoriasis
B Atopic eczema
C Dermatitis herpetiformis
D Lichen planus
E Seborrhoeic dermatitis
A

C

Dermatitis herpetiformis (C) is a chronic blistering skin condition associated with gluten intolerance or coeliac disease. The itchy papulovesicular eruptions are distributed symmetrically on extensor surfaces. Gluten intake will exacerbate the symptoms.

Psoriasis (A) affects 2% of the UK population and also presents on the extensor aspects of limbs, and on nails and scalp. The most common form is psoriasis vulgaris, presenting with macules and papules covered with silvery scales (characteristic parakeratosis) and pinpoint bleeding. It is associated with arthritis. There is also a loss of granular layer, clubbing of rete ridges and Munro’s microabscesses. Ezcema’s histopathology is spongiosis of epidermis and perivascular chronic inflammatory infiltrate in the dermis. Acanthosis (epidermal thickening) would develop if eczema becomes chronic.

Atopic eczema (B) has a strong family history and association with asthma and not gluten sensitivity.

Seborrhoeic dermatitis (E) is a type of eczema that affects the apocrine areas of the face; sometimes known as the cradle cap in infants in severe forms (with lack of biotin causing a thick yellow crusty scalp rash).

Lichen planus (D) is the eruption of purple flat-topped papules affecting the skin, tongue and oral mucosa. There is characteris- tic hyperkeratosis with saw toothing of rete ridges and basal cell degen- eration with a lichenoid chronic inflammatory infiltrate.

64
Q
  1. Types of fractures
    A 26-year-old man presents to accident and emergency having fallen off his skateboard and landed with a big impact on his right side. His X-ray shows a fracture in the midshaft of his right humerus that appears splintered although the soft tissue is intact. What type of fracture is this?
A Greenstick fracture
B Transverse fracture
C Compound fracture
D Impacted fracture
E Comminuted fracture
A

E

Comminuted fractures (E) are also known as segmental fractures whereby the bone is splintered and a number of pieces are visible but with intact soft tissue.

A greenstick fracture (A) is specifically a paediatric problem where the fracture is transverse, but only partially, and therefore the bone bends away from the long axis but remains attached.

A transverse fracture (B) is a fracture that is at right angles to the bone’s long axis and may be partial or complete (cortex to cortex). It is often a simple clean break with intact soft tissue.

A compound fracture (C) is when the fracture penetrates the skin surface and it is also known as an open fracture. These pose a very serious risk of infection due to contamination of the wound and exposed fracture.

Impacted fracture (D) is when the fracture site is ‘crushed’ inwards.

65
Q
  1. Non-neoplastic bone disease
    An 80-year-old woman presents complaining of pain on movement and stiffness after inactivity in her legs, most notably in her hips and knees. She also complains of pain in her hands and marked symmetrical swelling is noted in her distal interphalangeal joints. The X-ray of her right knee shows subchondral sclerosis, subchondral cyst formation, joint space narrowing and osteophytes. What is the most likely diagnosis?
A Osteoarthritis
B Rheumatoid arthritis
C Ankylosing spondylitis
D Psoriatic arthritis
E Osteoporosis
A
A
Osteoathritis (A) affects mainly the vertebrae, hips, knees, distal interphalangeal joints, carpometacarpal and metatarsophalangeal joints. The presentation of this patient is typical of osteoarthritis. 

Rheumatoid arthritis (B) is severe chronic relapsing synovitis that often presents in a younger population than this patient (30–40 years of age). 80% of patients are rheumatoid factor positive and characteristic deformities include ulnar deviation of the fingers, swan-neck and Boutonniere deformity of the fingers and a ‘Z’-shaped thumb. It characteristically spares the distal interphalangeal joint and affects symmetrically the small joints in the hands and feet, wrists, elbows, ankles and knees. Histopathology shows proliferative synovitis with thickening of the synovial membranes, hyperplasia of surface synoviocytes, intense inflammatory cell infiltrate and fibrin deposition and necrosis.

Ankylosing spondylitis (C) is a chronic inflammatory arthritis with strong HLA B27 association affecting the spine and sacroilium that can cause eventual fusion of the spine (bamboo spine). It commonly presents in a male aged 20–40 with stiffness of the spine.

In addition to joint involvement, psoriatic arthritis (D) will commonly present with psoriatic nail lesions (pitting, onycholysis), dactylitis and tendinitis. X-rays will show ‘new fluffy’ bone.

Osteoporosis (E) would show osteopenia, cortical thinning and increased radiolucency in the X-rays.

66
Q
  1. Painful joint
    A 59-year-old man presents to accident and emergency with a painful, swollen and hot big toe. The joint aspirate shows negatively birefringent crystals under polarized red light. The crystals are needle shaped. What is the most likely diagnosis?
A Pseudogout
B Lyme disease
C Reiter’s sydrome
D Gout
E Osteomyelitis
A

D

Only 2 conditions out of the 5 cause crystal-induced arthritis: gout and pseudogout. Gout (D) affects the big toe in 90% of cases and needle-shaped crystals occur in the joint causing very severe pain. One might also see tophi, which are of monosodium glutamate precipitate in pathognoic for gout where monosodium urate crystals are deposited under the skin. Monosodium urate crystals are negatively bi-refringent when viewed using a polarising microscope.

Pseudogout (A) is the calcium crystal deposition disease, either calcium pyrophosphate or calcium phosphate (hydroxyapatite). Chondrocalcinosis is seen on X-rays and the most commonly affected joint is the knee. On microscopic investigation with crossed polarizing filters of joint fluid obtained by aspiration, rhombus-shaped, positively birefringent crystals are seen.

Lyme disease (B) is an inflammatory arthropathy resulting from a tick bite, caused by Borrelia burgdorferi. The associated skin rash is erythema chronicum migrans and arthritis develops only in advanced disease.

Reiter’s syndrome (C) characteristically presents following an infectious episode with the triad reactive arthritis, uveitis and urethritis. Reactive arthritis is a rheumatoid factor-seronegative, HLA B27-linked spondyloarthopathy commonly affecting the larger joints (e.g. sacroiliac and knee joints). Patients also sometimes develop circinate balanitis, keratoderma blenorrhagica and enthesitis of the Achilles tendon (causing heel pain).

Osteomyelitis (E) in the big toe in adults is often secondary to diabetic skin ulcer, which this patient does not have.

67
Q
  1. Renal disease
    A 23-year-old man presents to accident and emergency with a 2-day history of left-sided loin pain, fever, rigors and vomiting. Urine analysis reveals microscopic haematuria and white cell casts. What is the most likely diagnosis?
A Cystitis
B Prostatitis
C Urolithiasis
D Acute pyelonephritis
E Hydronephrosis
A

D

Acute pyelonephritis (D) is strongly indicated if white cell casts are present. Red cell casts are strongly suggestive of glomerulonephritis. Eosinophiluria is strongly suggestive of tubulointerstitial nephritis.
Most cases of pyelonephritis begin as lower urinary tract infections, including cystitis (A) and prostatitis (B), but given the pain distribution and presentation of this patient, it is much more likely to be infection of the left kidney (pyelonephritis). 

Urolithiasis (C) can cause the pain and haematuria present in this patient but it does not explain the fever and rigors as this condition is not caused by infection but by four types of calculi: calcium, triple phosphate/struvite, uric acid and cystine. However, these are likely to predispose to infection.

Hydronephrosis (E) can present acutely with sudden onset severe pain or a more chronic presentation as episodes of dull discomfort. Nausea and vomiting may occur but it is unlikely to cause fever or rigors unless the blocking of urine flow results in a urinary tract infection.

68
Q
  1. Anti-topoisomerase antibodies
    A 35-year-old woman presents to accident and emergency with nausea, severe malaise, swelling and stiffness of the fingers. On examination, her blood pressure is 155/95mmHg and she has Raynaud’s phenomenon. Blood tests reveal positive anti-topoisomerase antibodies and deranged serum creatinine and urea. A biopsy result of her small arteries reveals an onion skin appearance. What is the most likely diagnosis?
A Systemic lupus erythematosus
B Diffuse scleroderma
C Kawasaki’s disease
D Polyarteritis nodosa
E Limited scleroderma/CREST
A

B

Scleroderma exists as limited and diffuse forms. This patient has internal involvement with the evidence of renal failure, which is consistent with diffuse scleroderma (B), which is a rapidly progressing condition that affects a large area of skin and one or more internal organs such as the kidneys, oesophagus and heart.

In limited scleroderma (E) there are predominantly cutaneous manifestations affecting the hands, arms and face associated with Calcinosis, Raynaud’s phenomenon, Esophaegeal dysfunction, Sclerodactyly and Telangiectasias, i.e. CREST syndrome. The immune system abnormalities in scleroderma are: decreased CD8 cells, antibodies to DNA topoisomerase (in the diffuse form only) and anti-centromere antibody (in limited form). The pathology underlying scleroderma is fibrosis with the unique ‘onion skin’ intimal thickening of small arteries, which is due to arteriolosclerosis.

The remaining three options are unlikely for the following reasons: systemic lupus erythematosus (A) often presents with the characteristic photosensitive butterfly malar rash on the face and antibodies to double-stranded DNA.

Polyarteritis nodosa (D) is the inflammatory necrosis of the walls of small and medium-sized arteries. It is usually associated with chronic hepatitis B infection and the presence of anti-neutrophil cytoplasmic antibodies (ANCA).

Kawasaki disease (C) is an autoimmune condition that largely affects children under 5 years of age.

69
Q
  1. Cerebrovascular pathology
A Subarachnoid haemorrhage
B Parkinson’s disease
C Extradural haemorrhage
D Vascular dementia
E Subdural haemorrhage
F Intracerebral haemorrhage 
G Multiple sclerosis
H Duret haemorrhage
I Alzheimer’s disease

1 A 54-year-old man is seen in the neurology clinic due to tremor and rigidity. A DAT scan reveals reduced uptake in the substantia nigra.

A

1) B

Parkinson’s disease (B) is a degenerative disorder associated with basal ganglia dysfunction. Clinical features can be remembered by the mnemonic SMART: shuffling gait, mask-like-face, akinesia, rigidity and tremor. Degeneration of the substantia nigra and locus coeruleus of the basal ganglia leads to reduced production of dopamine. At the microscopic level, inclusion bodies known as Lewy bodies are deposited in the cytoplasm of neurons that are made up of alpha-synuclein. Parkinson’s disease may be associated with Lewy body dementia.

Extradural haemorrhage (C) is a bleed above the dura due to damage to the middle meningeal artery. It is usually associated with severe trauma and fracture to the pterion and a subsequent lucid interval.

Vascular dementia (D) often progresses in a step-wise manner. It is associated with cardiovascular risk factors such as hypertension, smoking and diabetes mellitus.

Intracerebral haemorrhage (F) cause approximately 20 per cent of strokes and is perpetuated by hypertension and arteriovenous malformations. Hypertension may lead to either hyaline arteriosclerosis or Charcot–Bouchard aneurysms.

Duret haemorrhage (H) is associated with tentorial (medial temporal lobe) and tonsillar (cerebellar tonsil) herniation, due to tearing of vessels. Herniation may occur due to raised intracranial pressure.

70
Q
  1. Cerebrovascular pathology
A Subarachnoid haemorrhage
B Parkinson’s disease
C Extradural haemorrhage
D Vascular dementia
E Subdural haemorrhage
F Intracerebral haemorrhage 
G Multiple sclerosis
H Duret haemorrhage
I Alzheimer’s disease

2 A 74-year-old man presents to accident and emergency with increasing headache and confusion. The man’s wife suggests her husband may have tripped and fallen 3 days previously.

A

2) E

Subdural haemorrhage (E) occurs between the dura and arachnoid due to an acute tear in bridging veins. This tends to occur after a clear history of trauma. Bleeding results in features of raised intracranial pressure. As the bleeding is venous in nature, haematoma development is slow (usually taking 48 hours) and as a result raised intracranial pressure takes time to become apparent. Chronic subdural haemorrhage refers to a re-bleed of a previous bridging vein subdural haemorrhage. Patients will usually present with an altered mental state.

Extradural haemorrhage (C) is a bleed above the dura due to damage to the middle meningeal artery. It is usually associated with severe trauma and fracture to the pterion and a subsequent lucid interval.

Vascular dementia (D) often progresses in a step-wise manner. It is associated with cardiovascular risk factors such as hypertension, smoking and diabetes mellitus.

Intracerebral haemorrhage (F) cause approximately 20% of strokes and is perpetuated by hypertension and arteriovenous malformations. Hypertension may lead to either hyaline arteriosclerosis or Charcot–Bouchard aneurysms.

Duret haemorrhage (H) is associated with tentorial (medial temporal lobe) and tonsillar (cerebellar tonsil) herniation, due to tearing of vessels. Herniation may occur due to raised intracranial pressure.

71
Q
  1. Cerebrovascular pathology
A Subarachnoid haemorrhage
B Parkinson’s disease
C Extradural haemorrhage
D Vascular dementia
E Subdural haemorrhage
F Intracerebral haemorrhage 
G Multiple sclerosis
H Duret haemorrhage
I Alzheimer’s disease

3 A 45-year-old woman presents to accident and emergency with the worst headache she has ever experienced. She is noted to have polycystic kidney disease.

A

3) A

Subarachnoid haemorrhage (A) occurs in the subarachnoid space. Potential causes include a saccular ‘berry’ aneurysm (most commonly occurring at artery bifurcations of the anterior circulation), hypertension, trauma, arteriovenous malformations and coagulation disorders. Clinical features include a severe ‘thunder clap’ headache radiating to the occiput; this may be preceded by a warning bleed causing a sentinel bleed. Subarachnoid haemorrhages are more commonly associated with polycystic kidney disease, coarctation of the aorta and fibromuscular dysplasia.

Extradural haemorrhage (C) is a bleed above the dura due to damage to the middle meningeal artery. It is usually associated with severe trauma and fracture to the pterion and a subsequent lucid interval.

Vascular dementia (D) often progresses in a step-wise manner. It is associated with cardiovascular risk factors such as hypertension, smoking and diabetes mellitus.

Intracerebral haemorrhage (F) cause approximately 20% of strokes and is perpetuated by hypertension and arteriovenous malformations. Hypertension may lead to either hyaline arteriosclerosis or Charcot–Bouchard aneurysms.

Duret haemorrhage (H) is associated with tentorial (medial temporal lobe) and tonsillar (cerebellar tonsil) herniation, due to tearing of vessels. Herniation may occur due to raised intracranial pressure.

72
Q
  1. Cerebrovascular pathology
A Subarachnoid haemorrhage
B Parkinson’s disease
C Extradural haemorrhage
D Vascular dementia
E Subdural haemorrhage
F Intracerebral haemorrhage 
G Multiple sclerosis
H Duret haemorrhage
I Alzheimer’s disease

4 A 35-year-old woman presents to the neurology clinic with weakness of her left side. On examination she is found to have nystagmus and an intention tremor. The patient complains of blurred vision for the past month.

A

4) G

Multiple sclerosis (MS; G) is a demyelinating disease of the upper motor system which follows a relapsing and remitting course. Histological features along the central nervous system include active (contain lymphocytes and macrophages) and inactive plaques (reduced nuclei and myelin). Clinical features include optic neuritis, intranuclear opthalmoplegia (disruption of medial longitudinal fasciculus) and cerebellar signs, as well as spasticity and weakness of limbs. Variants of MS include Devic disease (a more aggressive form) and Marburg MS (a fulminant form).

Extradural haemorrhage (C) is a bleed above the dura due to damage to the middle meningeal artery. It is usually associated with severe trauma and fracture to the pterion and a subsequent lucid interval.

Vascular dementia (D) often progresses in a step-wise manner. It is associated with cardiovascular risk factors such as hypertension, smoking and diabetes mellitus.

Intracerebral haemorrhage (F) cause approximately 20 per cent of strokes and is perpetuated by hypertension and arteriovenous malformations. Hypertension may lead to either hyaline arteriosclerosis or Charcot–Bouchard aneurysms.

Duret haemorrhage (H) is associated with tentorial (medial temporal lobe) and tonsillar (cerebellar tonsil) herniation, due to tearing of vessels. Herniation may occur due to raised intracranial pressure.

73
Q
  1. Cerebrovascular pathology
A Subarachnoid haemorrhage
B Parkinson’s disease
C Extradural haemorrhage
D Vascular dementia
E Subdural haemorrhage
F Intracerebral haemorrhage 
G Multiple sclerosis
H Duret haemorrhage
I Alzheimer’s disease

5 A 42-year-old man who suffers from Down syndrome is brought to see his GP by his carer. The carer describes how the patient has been wandering out of the house with increased frequency as well as becoming uncharacteristically aggressive, especially in the evening.

A

5) I

Alzheimer’s disease (I) is a progressive degenerative disease which mainly occurs in patients over the age of 50 years and the condition is most commonly sporadic. In some instances, there may be a genetic component such as the amyloid precursor protein as well as presenelins 1 and 2 mutations associated with Down syndrome. Inheritance of the e4 allele of apolipoprotein E increases risk of developing Alzheimer’s disease. Histological features include vascular wall deposition of Beta-amyloid (amyloid angiopathy), neurofibrillary tangles and neuritic plaques.

Extradural haemorrhage (C) is a bleed above the dura due to damage to the middle meningeal artery. It is usually associated with severe trauma and fracture to the pterion and a subsequent lucid interval.

Vascular dementia (D) often progresses in a step-wise manner. It is associated with cardiovascular risk factors such as hypertension, smoking and diabetes mellitus.

Intracerebral haemorrhage (F) cause approximately 20 per cent of strokes and is perpetuated by hypertension and arteriovenous malformations. Hypertension may lead to either hyaline arteriosclerosis or Charcot–Bouchard aneurysms.

Duret haemorrhage (H) is associated with tentorial (medial temporal lobe) and tonsillar (cerebellar tonsil) herniation, due to tearing of vessels. Herniation may occur due to raised intracranial pressure.

74
Q
  1. Urological neoplasia

4yo boy presents with a large abdominal mass and haematuria. BP is 165/120mmHg. The mass has a large necrotic solid tumour with extrarenal invasion. Microscopically, there are immature-looking glomerular structures. Aggressive therapy with surgery, chemotx and radiotx is indicated. What is the most likely diagnosis?

A Teratoma
B Wilm’s tumour
C Oncocytoma
D Spermatocytic seminoma
E Bowen’s disease
A

B
Wilm’s tumour (B) (nephroblastoma) is a malignant embryonic tumour derived from the primitive metanephros. It is the most common childhood urological malignancy and involves mutations of the tumour suppressor gene WT1 located on chromosome 11. Macroscopically, the kidney is replaced by rounded masses of solid, fleshy, white lesions with vast amounts of necrosis. Microscopically, it is composed of four elements: immature-looking glomerular structures, primitive small cell blastaematous tissue, epithelial tubules and stroma composed of spindle cells and striated muscle.

Teratoma (A) is a tumour of germ cell origin involving the endoderm, ectoderm and mesoderm, with a peak age of onset of 20–30 years.

Oncocytoma (C) is a benign epithelial tumour composed of large cells with granular, eosinophilic cytoplasm filled with mitochondria. It has little necrosis or haemorrhage. It is a variant of renal adenoma and is often confused with renal cell carcinoma.

Spermatocytic seminoma (D) is a germ cell malignant tumour of the testes, with a peak age between 30 and 40 years of age. Histopathology shows large tumour cells with small cells resembling spermatocytes.

Bowen’s disease (E) is one of two types of penile carcinoma in situ, often associated with human papillomavirus (HPV) infection and clinically appears as opaque plaques with shallow ulcerations.

75
Q
  1. Gleason’s grading system

A 79-year-old man has hesitancy and terminal dribbling urinary symptoms secondary to a tumour growth. No other symptoms are present. PR: prostate is reported as hard and craggy. The patient has been given a Gleason’s score of 8; a primary grade of 3, describing that the tissue has recognizable glands and these cells are beginning to invade the surrounding tissue. There is also a secondary grade of pattern 5 suggesting poorly differentiated cells.

What is the most likely diagnosis?

A Prostatic adenocarcinoma
B Seminoma
C Prostatic intraepithelial neoplasia
D Benign prostatic hyperplasia
E Transitional cell carcinoma
A

A

Prostatic adenocarcinoma (A) is rare in men under 50 years of age. The gross histopathology involves the peripheral portion of prostate, posteriorly. It is poorly demarcated, firm and yellow and there may be urinary bladder, seminal vesicle and rectal invasion. The Gleason’s scoring system is used to help evaluate the prognosis of men with prostate cancer. The pathologist assigns a grade to the most common tumour pattern and a second grade to the worst pattern, then the two grades are added together to get the Gleason score.

Seminoma (B) is the most common malignant testicular tumour and often presents as a painless swelling.

Prostatic intraepithelial neoplasia (C) is a common finding in younger men and poses an increased risk for developing carcinoma.

Benign prostatic hyperplasia (D) is a non- neoplastic enlargement of the prostate, which is very common and affects almost all men >70 years of age. The treatment for benign prostatic hyperthrophy is different to malignant disease, and includes alpha blockers, and drugs to reduce intra-prostatic dihydrotestosterone. Less radical surgery such as TURP is commonly used.

Transitional cell carcinoma (E) can arise anywhere in the bladder, with a significant association to smoking and chemical exposure. Clinically, there should be evidence of haematuria, previous cystitis and obstruction.

76
Q
  1. Non-neoplastic disorders of the breast
    A 27-year-old lactating mother presents with a painful red left breast. On closer examination, there are cracks and fissures on the left nipple. What is the most likely diagnosis?
A Fat necrosis
B Acute mastitis
C Duct ectasia
D Simple fibrocystic change
E Epithelial hyperplasia
A

B

Acute mastitis (B) is the acute inflammation of the breast that often affects lactating women. The usual causative organism is Staphylococcus aureus. An abscess may develop and drainage and antibiotics are curative.

Fat necrosis (A) is an inflammatory reaction to trauma of the adipose tissue in the breast and histopathology shows necrosis with multinucleated giant cells and later fibrosis. It may present as a discrete firm lump mimicking a carcinoma.

Duct ectasia (C) usually presents with nipple discharge and is due to inflammation and dilation of the large breast ducts. There may also be a breast mass and nipple retraction.

Simple fibrocystic change (D) is not an inflammatory disorder but a fibrocystic disorder. It is very common and presents with breast lumpiness and nodularity which may be cyclical, representing exaggerated hormonal responses.

Epithelial hyperplasia (E) is also a fibrocystic disorder that describes epithelial cell proliferation within the ducts or lobules in the breast, which increases risk of developing malignancy. It can only be diagnosed by histopathology.

77
Q
  1. Neoplasms of the breast (1)
    A 56-year-old woman presents with blood stained nipple discharge and a solitary mass located just superior to the nipple in her left breast. A histopathology analysis shows that a papillary mass is lined by epithelium and myoepithelium. It is believed that there is no increased risk of malignancy. What is the most likely diagnosis?
A Intraductal papilloma
B Phylloides tumour
C Fibroadenoma
D Radial scar
E Ductal carcinoma in situ
A

A

Intraductal papilloma (A) is a benign papillary tumour arising within the duct system of the breast. The two forms are peripheral (arise within small terminal ductules) and central (arise in larger lactiferous ducts). Only central papillomas can present with nipple discharge that is blood stained, while peripheral papillomas can remain clinically silent.

A fibroadenoma (C) is a benign fibroepithelial neoplasm of the breast that is common and presents as a circumscribed mobile breast lump in younger women (aged 20–30).

Phylloides tumours (B) are a group of potentially aggressive fibroepithelial neoplasms of the breast, they are uncommon and present as enlarging masses in women aged over 50. Some could have been pre-existing fibroadenomas.

A radial scar (D) is a benign sclerosing lesion characterized by a central zone of scarring surrounded by a radiating zone of proliferating glandular tissue. They are common and usually present as stellate masses on screening mammograms.

Ductal carcinoma in situ (E) has an inherent risk of progression to invasive breast carcinoma. Only 10 % produce clinical features such as a lump, nipple discharge or an eczematous change of the nipple (Paget’s disease of the nipple).

78
Q
  1. Neoplasms of the breast (2)
    A 53-year-old overweight woman with a positive family history of breast cancer attends her appointment for the NHS Breast Screening Programme. She is one of the 5 per cent of women who have an abnormal mammogram and are called for a core biopsy. She has been given a B code of B5b. What is the most likely diagnosis?
A Benign abnormality
B Lesion of uncertain malignant potential
C Ductal carcinoma in situ
D Invasive carcinoma
E Suspicious of malignancy
A

D

Invasive breast carcinomas (D) are a group of malignant epithelial tumours which infiltrate within the breast and have capacity to spread to distant sites.

RFs: early menarche, late menopause, increased weight, high alcohol consumption, oral contraceptive use and a positive family history. BRCA mutations are known to cause a life-time risk of invasive breast carcinoma of up to 85%. The types of invasive carcinomas include: invasive ductal carcinoma, invasive lobular carcinoma, mucinous, tubular, medullary and papillary. Ductal carcinoma in situ (DCIS) is a form of non-invasive breast carcinoma.

The screening programme in the UK is a 3-yearly mammography for all women between 50 and 70 years of age. Those with an abnormal mammogram (5%) are recalled for further investigation, which may include more mammograms or an ultrasound followed by sampling of the abnormal area, usually by core biopsy.

The histology core biopsy codes are:
   B1 = normal breast tissue.
   B2 = benign abnormality (A)
   B3 = lesion of uncertain malignant potential (B)    
   B4 = suspicious of malignancy (E)
   B5 = malignant
   B5a = ductal carcinoma in situ (C)
   B5b = invasive carcinoma
79
Q
  1. Cystic diseases of the kidneys
    A 44-year-old man has developed end stage renal failure over the past 5 years with numerous episodes of macroscopic haematuria. He was asymptomatic previously. Ultrasound scan has shown numerous asymmetrical large cysts bilaterally. The patient’s mother had a similar condition. What is the most likely diagnosis?
A Acquired cystic disease
B Medullary sponge disease
C Adult polycystic kidney disease
D Cystic renal dysplasia
E Simple renal cysts
A

C

Adult polycystic kidney disease (C) is an autosomal dominant condition, believed to involve 2 genes: PKD1 and PKD2. Both kidneys are replaced by fluid-filled cysts. It is asymptomatic initially, but in the fourth decade or later, the patient presents with a large lobulated abdominal mass, pain or haematuria. It is managed by blood pressure control and eventually dialysis or transplantation.

Acquired cystic disease (A) occurs in patients who are an dialysis and therefore inconsistent with this clinical picture. That does not mean that dialysis causes cysts. The cause is, in fact, not clear, but it seems that dialysis does not clear a circulating factor that causes such cysts. Without dialysis, these individuals would not survive, which is why patients on dialysis for long periods are those who have such cysts.

Medullary sponge disease (B) is occasionally a familial condition that does not impair renal function and presents with renal stones predispos- ing to renal colic and infections.

Cystic renal dysplasia (D) is a failure of differentiation of metanephric tissues, it is typically asymptomatic and it is not hereditary in nature but developmental, therefore often presenting in childhood.

Simple renal cysts (E) are lesions in otherwise normal kidneys that can be solitary or occasionally multiple. They are typically asymptomatic and are acquired.

80
Q
  1. Glomerular disease
    A 48-year-old man presents with oliguria and a vasculitic rash on his legs. Investigations indicate that he has a reduced glomerular filtration rate and urinalysis finds urine casts containing red and white blood cells. Histopathology shows scanty deposits of immunoglobulins and complement present with associated anti-neutrophil cytoplasm antibodies (ANCA). What is the most likely diagnosis?

A IgA nephropathy
B Thrombotic microangiopathy
C Anti-GBM crescentic glomerulonephritis disease
D Pauci-immune crescentic glomerulonephritis disease
E Amyloidosis

A

D

Pauci-immune crescent glomerulonephritis (D) is associated with anti- neutrophil cytoplasm antibodies (ANCA) and vasculitis in other systems such as the skin and lungs. This patient has glomerulonephritis that is sufficient to cause acute renal failure and therefore is almost certain to be associated with glomerular crescents.

IgA nephropathy (A) is a cause of immune-complex crescentic glomerulonephritis, the key feature being immune complexes detected by immunohistochemistry and electron microscopy. IgA nephropathy is the most common form of glomerulonephritis worldwide, where there is a predominant IgA deposition in glomeruli, often present with microscopic or macroscopic haematuria.

The anti-GBM form (C) is characterized by linear localization of IgG on GBM by immunofluorescence and because antibodies bind to alveolar basement membranes, it often leads to lung haemorrhage.

Thrombotic microangiopathy (B) is where red blood cells may become damaged by fibrin leading to haemolysis (microangiopathic haemolytic anaemia). There are two forms: diarrhoea associated and non-diarrhoeal.

Amyloidosis (E) is a systemic disease that can cause glomerular lesions, where the amyloid, an extracellular fibrillar protein can deposit. It can cause proteinuria, nephritic syndrome and chronic renal failure.

81
Q
  1. Clinical manifestations of glomerular disease
    A 35-year-old oedematous woman is found to have urinary protein loss of 5.1g daily. Further tests show a low albumin level and significant interference with podocyte function. No glomerular crescents were detected. What is the most likely diagnosis?
A Acute glomerulonephritis
B Nephritic syndrome
C Nephrotic syndrome
D Acute tubular necrosis
E Acquired cystic disease
A
C
Nephrotic syndrome (C) is the breakdown of selectivity of the glomerular filtration barrier leading to massive protein leak. There is proteinuria at least of 3.5g/day, hypoalbuminaemia, oedema and hyperlipidaemia. There are systemic and primary glomerular diseases. The latter interferes with podocyte function and can be one of 3 types: minimal change disease, focal segmental glomerulosclerosis (FSGS) and membranous glomerulonephritis.
Acute glomerulonephritis (A) is the acute inflammation of glomeruli leading to reduction in glomerular filtration rate. Characteristic presentation is with oliguria and urine casts containing red and white
blood cells. A severe form can cause renal failure and is almost always associated with glomerular crescents. Crescents are accumulations of proliferating epithelial cells in Bowman’s space and indicate destroyed glomeruli from which there can be no recovery. 

Nephritic syndrome (B) is characterized by haematuria, oedema, oliguria resulting in severe glo- merular damage and red blood cell leakage.

Acute tubular necrosis (D) is an acute, reversible renal failure caused by necrosis of renal tubular epithelial cells and therefore is a disease of the tubules and interstitium and not glomerular disease.

Acquired cystic disease (E) is not glomerular disease and also commonly arises in kidneys in situ when the patient is on dialysis for chronic renal failure.

82
Q
  1. Cerebrovascular aneurysm
    A 58-year-old woman is known to have a Berry aneurysm in the basilar artery. She develops sudden onset severe headache, nausea and loss of consciousness. There was evidence of a ‘warning bleed’ but no history of brain trauma. What is the most likely fatal diagnosis caused by the ruptured Berry aneurysm?
A Intracerebral haemorrhage
B Subarachnoid haemorrhage
C ‘Watershed’ strokes
D Transient ischaemic attack
E Tonsillar brain herniation
A

B

Subarachnoid haemorrhage (B) is usually non-traumatic and due to rupture of a Berry aneurysm, which are present in 1% of the general population. They occur on the circle of Willis, mostly at the arterial bifurcations of the internal carotid artery and within the vertebrobasilar circulation. The Berry aneurysms enlarge with time and pose the greatest risk of rupture when at 6–10mm diameter. They are more common in females than males and in 20% of cases the patients have a ‘warning bleed’, which will lead to a rupture and poses a much worse prognosis than without a ‘warning bleed’.

The basilar artery is an extracerebral artery and this rules out an intracerebral haemorrhage (A).

‘Watershed’ strokes (C) are infarctions caused by hypoperfusion at the periphery of a blood supply. There does not have to be an occlusion.

Transient ischaemic attacks (D) often precede the main event, lasting several minutes to 24h and caused by self-limiting vascular obstruction due to atheromatous emboli and/or platelet–fibrin aggregates. One third of patients with a TIA get significant infarct within five years.

Tonsillar brain herniation (E) is when cerebellar tonsils move downwards through the foramen magnum, this causes brainstem compression and is life threatening and often associated with secondary ‘Duret’ haemorrhage.

83
Q
  1. Cerebral infections
    A 9-year-old boy presents with fever, headache, stiff neck and altered mental state. His cerebrospinal fluid is turbid and contains mostly neutrophils. The meninges appear congested and there is purulent material in the subarachnoid space. What is the most likely causative organism?
A Coxsackie virus
B Treponema pallidum
C Staphylococcus aureus
D Streptococcus pneumoniae
E Haemophilus influenzae
A

D
The diagnosis is acute purulent meningitis. It is a major cause of morbidity and mortality in all ages, but in children older than 6 years, Streptococcus pneumoniae (D) is the most common cause. It used to
be Haemophilus influenzae (E) but incidence is falling due to vaccination. In neonates, the cause is usually the flora of the maternal genital tract, and the most likely organism would be group B Streptococcus and Escherichia coli. Neisseria meningitidis is the major cause of epidemics of meningitis in older children, adolescents and young adults and can present without a rash, although the presence of a non-blanching rash makes Neisseria a likely diagnosis, and this form of meningitis is rapidly fatal.

Staphylococcus aureus (C) and Gram-negative rods are common causes in people with surgical shunts. Meninges appear congested and purulent material is present in the subarachnoid space. Prognosis depends on speedy accurate treatment.

If the organism is viral, the diagnosis will be acute lymphocytic meningitis. This can be caused by the Coxsackie’s virus (A), echovirus, HIV and the mumps virus. The cerebrospinal fluid would contain mostly lymphocytes. Clinical features would often be less severe despite similar symptoms and signs to acute purulent meningitis.

Treponema pallidum (B) is a cause of chronic meningitis and the infiltrate would characteristically contain lymphocytes, plasma cells and epithelioid macrophages.

84
Q
  1. Gastrointestinal pathology
A Ulcerative colitis
B Chronic gastritis
C Oesophageal cancer
D Coeliac disease
E Gastric carcinoma
F Barrett’s oesophagus 
G Gardener’s syndrome 
H Crohn’s disease
I Peptic ulcer disease

1 A 35-year-old man has a 3-week history of bloody stools without mucus with associated weight loss. A biopsy of the gastrointestinal tract reveals non- caseating granulomas with transmural inflammation.

A

1) H

Crohn’s disease (H) is an inflammatory bowel disease which can affect any section of the gastrointestinal system. Characteristics include transmural inflammation (full intestinal wall thickness), skip lesions and fistulae. Biospy of the gastrointestinal wall will reveal non-caseating granulomas in approximately 60% of cases. Complications include thickening of the bowel wall (also known as a ‘rubber hose wall’) which can lead to bowel obstruction. Extra-intestinal manifestations of Crohn’s disease include arthritis, ankylosing spondylosis, stomatitis and uveitis, as well as dermatological lesions (pyoderma gangrenosum and erythema nodosum).

Ulcerative colitis (A) is an inflammatory bowel disease, which differs from Crohn’s disease as inflammation is confined to the colon, superficial and continuous.

Chronic gastritis (B) may be caused by Helicobacter pylori or autoimmune disease. Histological features include lymphocyte infiltrate and intestinal metaplasia. There is a long-term risk of carcinoma or MALT lymphoma.

Oesophageal cancer (C) exists as squamous cell carcinoma (90%) primarily caused by smoking or alcohol, and adenocarcinoma (10%) caused by Barrett’s oesophagus.

Gardener’s syndrome (G) is similar to familial adenomatous polyposis (presence of adenomas in the gastrointestinal tract caused by a defect in the APC gene), with the addition of extra-intestinal growths (osteomas, epidermoid cysts and desmoid tumours).

85
Q
  1. Gastrointestinal pathology
A Ulcerative colitis
B Chronic gastritis
C Oesophageal cancer
D Coeliac disease
E Gastric carcinoma
F Barrett’s oesophagus 
G Gardener’s syndrome 
H Crohn’s disease
I Peptic ulcer disease

2 A 24-year-old woman presents to her GP with a 2-week history of diarrhoea, weight loss and fatigue. Biopsy of the gastrointestinal tract reveals villous atrophy with crypt hyperplasia.

A

2) D

Coeliac disease (D) is an autoimmune disease that occurs due to gluten sensitivity, specifically gliadin found in wheat, barley and rye. It affects primarily the duodenum and jejunum. CD8+ cells are sensitized to gliadin; they accumulate in the gut and attack enterocytes and as a result villi disappear. As a compensatory mechanism immature enterocytes in the crypts proliferate causing deeper crypts. Therefore, features of gut biopsy samples include intraepithelial lymphocytes, villous atrophy and crypt hyperplasia. Clinical features include steatorrhoea, bloating, weight loss and fatigue (anaemia secondary to malabsorption).

Ulcerative colitis (A) is an inflammatory bowel disease, which differs from Crohn’s disease as inflammation is confined to the colon, superficial and continuous.

Chronic gastritis (B) may be caused by Helicobacter pylori or autoimmune disease. Histological features include lymphocyte infiltrate and intestinal metaplasia. There is a long-term risk of carcinoma or MALT lymphoma.

Oesophageal cancer (C) exists as squamous cell carcinoma (90%) primarily caused by smoking or alcohol, and adenocarcinoma (10%) caused by Barrett’s oesophagus.

Gardener’s syndrome (G) is similar to familial adenomatous polyposis (presence of adenomas in the gastrointestinal tract caused by a defect in the APC gene), with the addition of extra-intestinal growths (osteomas, epidermoid cysts and desmoid tumours).

86
Q
  1. Gastrointestinal pathology
A Ulcerative colitis
B Chronic gastritis
C Oesophageal cancer
D Coeliac disease
E Gastric carcinoma
F Barrett’s oesophagus 
G Gardener’s syndrome 
H Crohn’s disease
I Peptic ulcer disease

3 A 54-year-old man presents to his GP with a 2-week history of worsening dysphagia. The patient’s past medical history reveals severe gastro-oesophageal reflux disease. A duodenoscopy suggests metaplastic transformation of the lower oesophageal region.

A

3) F

Barrett’s oesophagus (F) occurs as a result of chronic gastro-oesophageal reflux disease. At the squamo-columnar junction between the lower oesophagus and stomach, squamous epithelial cells usually exist. Acidity causes transformation of squamous epithelial cells to columnar epithelial cells, a metaplastic change. The metaplastic columnar epithelial cells include goblet cells that produce intestinal mucin; hence the process is termed intestinal metaplasia. The major complication of Barrett’s oesophagus is an increased risk of adenocarcinoma of the oesophagus.

Ulcerative colitis (A) is an inflammatory bowel disease, which differs from Crohn’s disease as inflammation is confined to the colon, superficial and continuous.

Chronic gastritis (B) may be caused by Helicobacter pylori or autoimmune disease. Histological features include lymphocyte infiltrate and intestinal metaplasia. There is a long-term risk of carcinoma or MALT lymphoma.

Oesophageal cancer (C) exists as squamous cell carcinoma (90%) primarily caused by smoking or alcohol, and adenocarcinoma (10%) caused by Barrett’s oesophagus.

Gardener’s syndrome (G) is similar to familial adenomatous polyposis (presence of adenomas in the gastrointestinal tract caused by a defect in the APC gene), with the addition of extra-intestinal growths (osteomas, epidermoid cysts and desmoid tumours).

87
Q
  1. Gastrointestinal pathology
A Ulcerative colitis
B Chronic gastritis
C Oesophageal cancer
D Coeliac disease
E Gastric carcinoma
F Barrett’s oesophagus 
G Gardener’s syndrome 
H Crohn’s disease
I Peptic ulcer disease

4 A 45-year-old man is referred to the gastroenterology outpatient clinic due to severe epigastric pain and an episode of haematemesis. Further testing reveals he is Helicobacter pylori positive and has a 20 pack–year history of smoking.

A

4) I

Peptic ulcer disease (I) can either be duodenal or gastric. Ulcers differ from erosions as they extend to the submucosa (sometimes to the muscularis mucosa) and take weeks to heal, whereas erosions breach the mucosa only and take days to heal. The main causes of peptic ulcers are Helicobacter pylori, NSAIDs, Zollinger–Ellison syndrome and smoking. These factors disrupt the balance between protective (mucus layer and bicarbonate secretion) and damaging (acid and enzymes) elements leading to ulceration.

Ulcerative colitis (A) is an inflammatory bowel disease, which differs from Crohn’s disease as inflammation is confined to the colon, superficial and continuous.

Chronic gastritis (B) may be caused by Helicobacter pylori or autoimmune disease. Histological features include lymphocyte infiltrate and intestinal metaplasia. There is a long-term risk of carcinoma or MALT lymphoma.

Oesophageal cancer (C) exists as squamous cell carcinoma (90%) primarily caused by smoking or alcohol, and adenocarcinoma (10%) caused by Barrett’s oesophagus.

Gardener’s syndrome (G) is similar to familial adenomatous polyposis (presence of adenomas in the gastrointestinal tract caused by a defect in the APC gene), with the addition of extra-intestinal growths (osteomas, epidermoid cysts and desmoid tumours).

88
Q
  1. Gastrointestinal pathology
A Ulcerative colitis
B Chronic gastritis
C Oesophageal cancer
D Coeliac disease
E Gastric carcinoma
F Barrett’s oesophagus 
G Gardener’s syndrome 
H Crohn’s disease
I Peptic ulcer disease

5 A 56-year-old man presents to his GP with abdominal pain, weight loss and fatigue. A duodenoscopy allows a biopsy of a gastric lesion to be taken, which demonstrates signet ring cells and linitis plastica.

A

5) E

Gastric carcinoma (E) is usually a consequence of chronic gastritis and hence Helicobacter pylori is implicated in the pathogenesis. H. pylori causes intestinal metaplasia leading to gastric atrophy which becomes dysplasia and eventually carcinoma. On histology the carcinoma can be ulcer-like, but differs from peptic ulcers as they have irregular borders and raised edges. Features of gastric carcinoma are the presence of signet ring cells (cells with compressed nuclei) and linitis plastica (the stomach becomes thick and rigid resembling a leather bottle).

Ulcerative colitis (A) is an inflammatory bowel disease, which differs from Crohn’s disease as inflammation is confined to the colon, superficial and continuous.

Chronic gastritis (B) may be caused by Helicobacter pylori or autoimmune disease. Histological features include lymphocyte infiltrate and intestinal metaplasia. There is a long-term risk of carcinoma or MALT lymphoma.

Oesophageal cancer (C) exists as squamous cell carcinoma (90%) primarily caused by smoking or alcohol, and adenocarcinoma (10%) caused by Barrett’s oesophagus.

Gardener’s syndrome (G) is similar to familial adenomatous polyposis (presence of adenomas in the gastrointestinal tract caused by a defect in the APC gene), with the addition of extra-intestinal growths (osteomas, epidermoid cysts and desmoid tumours).

89
Q
  1. Muscle weakness

46yo woman presents with gradual muscle weakness in her neck and upper arms over the past 3 weeks. She is also said to have a purple ‘heliotrope’ rash on her upper eyelids, an erythematous scaling rash on her face and red patches on the knees. She has also experienced some weight loss. Blood tests reveal elevated skeletal muscle enzymes but electromyogram results were negative. What is the most likely diagnosis?

A Polymyositis
B Henoch–Schönlein purpura
C Dermatomyositis
D Kawasaki disease
E Sarcoidosis
A

C

Dermatomyositis (C) has the muscle components involved in polymyositis but in addition it is accompanied by periorbital oedema and a characteristic purple ‘heliotrope’ rash on the upper eyelids. Heliotrope is a pink/ purple colour that one only hears about in reference to dermatomyositis, but is actually the colour of the heliotrope flaver. It is also commonly present with an erythematous, scaling rash on the face, shoulders, upper arms and chest, with red patches over the knuckles, elbows and knees. Weight loss and arthralgia may also present.

Polymyositis (A) is char-acterized by weakness, pain and swelling of proximal limb muscles and facial muscles, often with ptosis and dysphagia. It is not associated with any skin presentations and often there are positive electromyogram results.

Kawasaki disease (D) is a paediatric condition that does not cause any muscle weakness. Henoch–Schönlein purpura (B) is a systemic vasculitic condition that typically presents with symptoms including palpable purpura, joint pains and abdominal pains. Impaired renal function is a fairly common complication of Henoch–Schönlein purpura, which this patient does not have.

Sarcoidosis (E) is common in the age and gender of this patient but primarily affects the lungs and lymph nodes. The key presenting complaint here is the muscle weakness and this would exclude sarcoidosis: http://emedicine.medscape.com/article/1170205-diagnosis

90
Q
  1. HIV in children
    A 22-year-old HIV-infected woman is pregnant with her second child. HIV has been transmitted perinatally to her first child. One of the most successful interventions to reduce vertical transmission of HIV during pregnancy to less than 1% is the use of combination anti-retroviral treatment, which ideally should reduce the viral load to?
A 50 copies/mL
B 800 copies/mL
C 1000 copies/mL
D 5000 copies/mL
E 10 000 copies/mL
A

A

Almost all new childhood HIV infections are due to mother-to-child (vertical) transmission; before (in utero), during childbirth (intra partum) or through breastfeeding. HIV viral load tests are reported as the number of HIV copies in a millilitre (copies/mL) of blood. A high viral load can be anywhere from 5000 to 10000 copies/mL. Initial, untreated and uncontrolled HIV viral loads can range as high as one million or more copies/mL. A low viral load is usually between 40 and 500 copies/mL and this result indicates that HIV is not actively reproducing and that the risk of disease progression is low.

The clinical features of HIV infection include: suppurative ear infections, enlarged lymph nodes, hepatomegaly, splenomegaly, easy bruising, enlarged parotids, oral thrush, herpes zoster infection, clubbing, anaemia, progressive encephalopathy, frequent nose bleeds, failure to thrive, severe pneumonia (TB, LIP, PCC), severe nappy rash, recurrent or persistent diarrhoea. Classes of anti-retrovirals currently used for children in Africa are: non-nucleoside reverse transcriptase inhibitors, nucleoside reverse transcriptase inhibitors, nucleotide reverse transcriptase inhibitors and protease inhibitors.

The comprehensive approach to preventing HIV infection in infants is:

  • Prevention of HIV in parents
  • Prevention of unintended pregnancies among HIV-infected women
  • Prevention of transmission from an HIV-infected woman to her infant
  • Care and support for HIV-infected women, their infants and their families
91
Q
  1. Lewy bodies
    A 43-year-old man presents with a rest tremor, slowness of voluntary movement and rigidity. It is reported that he has a mutation of the alpha-synuclein protein and he is free of Lewy bodies on histological examination. What is the most likely diagnosis?
A Familial Parkinson’s disease
B Alzheimer’s disease
C Multiple system atrophy
D Multiple sclerosis
E Idiopathic Parkinson’s disease
A

A

The alpha-synuclein protein is a major component of Lewy bodies. Its true function is unknown but its accumulation has a toxic effect on plasma membranes. In the rare cases of familial forms of Parkinson’s disease (A), there is a mutation in the gene coding for alpha-synuclein and this condition is free of Lewy bodies. Patients with these mutations have a worse prognosis with earlier onset and non-responsiveness to levodopa. The alpha-synuclein protein can aggregate to form insoluble fibrils in pathological conditions characterized by Lewy bodies, such as idiopathic Parkinson’s disease (E), Alzheimer’s disease (B) and multi- ple system atrophy (C) (ruling them all out for being inconsistent with the case). Idiopathic Parkinson’s disease is a degenerative disease with cell death of the dopamine-producing neurons of the substantia nigra, resulting in depigmentation. Furthermore, patients with this condition are alpha-synuclein and ubiquitin positive.

Multiple sclerosis (D) often presents between the ages of 20 and 40 and is pathologically characterized by demyelinating plaques. It is also more common in females. The remaining diseases often present in an older patient.

92
Q
  1. Liver pathology
A Cholangiocarcinoma
B Cirrhosis
C Alpha1-Antitrypsin deficiency
D Haemosiderosis
E Primary biliary cirrhosis
F Haemochromatosis
G Hepatocellular carcinoma
H Primary sclerosing cholangitis 
I Wilson’s disease

1 A 56-year-old man with previous history of HCV infection presents to A&E with jaundice. His wife notes that he has recently been bruising very easily. Ultrasound of the patient’s liver reveals irregular echogenicity demonstrating nodules.

A

1) B

Cirrhosis (B) is defined as the diffuse fibrosis of the liver with abnormal architecture characterized by nodules secondary to chronic hepatic disease. This can be sub-classified as micronodular (<3mm; usually alcoholic aetiology) or macronodular (>3mm; usually viral aetiology). Fibrosis results from stellate cell activation, which deposit collagen. Nodules represent proliferating hepatocytes that lack normal acinar structure and hence have a haphazard blood supply; this leads to shunt formation and portal HTN. Causes of cirrhosis include alcohol, HBV and HCV, primary biliary cirrhosis, haemochromatosis, Wilson’s disease and alpha1-antitrypsin deficiency.

Cholangiocarcinoma (A) is an adenocarcinoma of the bile ducts. RFs for the development of cholangiocarcinoma include primary sclerosing cholangitis, parasitic liver fluke infection and exposure to medical imaging contrast media.

Alpha 1-Antitrypsin deficiency (C) results in destruction of tissues due to the lack of inhibition of neutrophil proteases, resulting in emphysema in the lungs and cirrhosis.

Haemosiderosis (D) is defined as excess haemosiderin deposition due to an acquired cause (alcohol and blood transfusions). Unless severe, there is no architectural change and therefore no development of cirrhosis.

Primary sclerosing cholangitis (H) affects the large intra- and extra- hepatic ducts. Histological features include periductal fibrosis that even- tually invades the lumen causing concentric onion-ring fibrosis.

93
Q
  1. Liver pathology
A Cholangiocarcinoma
B Cirrhosis
C Alpha1-Antitrypsin deficiency
D Haemosiderosis
E Primary biliary cirrhosis
F Haemochromatosis
G Hepatocellular carcinoma
H Primary sclerosing cholangitis 
I Wilson’s disease

2 A 35-year-old man presents to his GP with his mother with signs of Parkinsonism (tremor, rigidity and slow movement) as well as recent changes in his behaviour.

A

2) I

Wilson’s disease (I) is an autosomal recessive condition that results from a mutation in the ATP7B gene leading to multi-organ copper accumulation. ATP7B is a protein that facilitates the transport of copper across cell membranes. Liver involvement will lead to cirrhosis, most commonly in children, whilst accumulation in the brain can lead to Parkinsonism, seizures and dementia. Psychological features include behavioural changes, depression and psychosis. Other organs affected include the eyes (Kayser–Fleischer rings), kidneys (renal tubular acido- sis) and heart (cardiomyopathy).

Cholangiocarcinoma (A) is an adenocarcinoma of the bile ducts. Risk factors for the development of cholangiocarcinoma include primary sclerosing cholangitis, parasitic liver fluke infection and exposure to medical imaging contrast media.

Alpha 1-Antitrypsin deficiency (C) results in destruction of tissues due to the lack of inhibition of neutrophil proteases, resulting in emphysema in the lungs and cirrhosis.

Haemosiderosis (D) is defined as excess haemosiderin deposition due to an acquired cause (alcohol and blood transfusions). Unless severe, there is no architectural change and therefore no development of cirrhosis.

Primary sclerosing cholangitis (H) affects the large intra- and extra-hepatic ducts. Histological features include periductal fibrosis that eventually invades the lumen causing concentric onion-ring fibrosis.

94
Q
  1. Liver pathology
A Cholangiocarcinoma
B Cirrhosis
C Alpha1-Antitrypsin deficiency
D Haemosiderosis
E Primary biliary cirrhosis
F Haemochromatosis
G Hepatocellular carcinoma
H Primary sclerosing cholangitis 
I Wilson’s disease

3 A 56-year-old woman is investigated by the hepatology team for decompensated liver disease. A liver biopsy sample stains blue with Perl’s Prussian blue stain.

A

3) F

Haemochromatosis (F) is an autosomal recessive condition that is due to a mutation in the HFE gene. The HFE protein regulates iron absorption that is stored as haemosiderin. Histological features of haemochromatosis include a golden-brown haemosiderin deposition in the parenchyma of many organs. Haemosiderin eventually leads to inflammation and subsequent fibrosis. Histological samples of affected tissue will stain blue with Perl’s Prussian blue. Organs affected include the liver (cirrhosis), pancreas (diabetes), skin (bronzed pigmentation), heart (cardiomyopathy) and gonads (atrophy and impotence).

Cholangiocarcinoma (A) is an adenocarcinoma of the bile ducts. Risk factors for the development of cholangiocarcinoma include primary sclerosing cholangitis, parasitic liver fluke infection and exposure to medical imaging contrast media.

Alpha 1-Antitrypsin deficiency (C) results in destruction of tissues due to the lack of inhibition of neutrophil proteases, resulting in emphysema in the lungs and cirrhosis.

Haemosiderosis (D) is defined as excess haemosiderin deposition due to an acquired cause (alcohol and blood transfusions). Unless severe, there is no architectural change and therefore no development of cirrhosis.

Primary sclerosing cholangitis (H) affects the large intra- and extra- hepatic ducts. Histological features include periductal fibrosis that even- tually invades the lumen causing concentric onion-ring fibrosis.

95
Q
  1. Liver pathology
A Cholangiocarcinoma
B Cirrhosis
C Alpha1-Antitrypsin deficiency
D Haemosiderosis
E Primary biliary cirrhosis
F Haemochromatosis
G Hepatocellular carcinoma
H Primary sclerosing cholangitis 
I Wilson’s disease

4 A 53-year-old man who has recently emigrated from sub-Saharan Africa is referred to the hepatology department due to recent onset weight loss, jaundice and ascites. There is history of previous aflatoxin exposure.

A

4) G

Hepatocellular carcinoma (HCC; G) is the most prevalent primary liver malignancy. Most commonly HCC occurs secondary to cirrhosis. The risk factors for HCC are therefore the numerous causes of cirrhosis. However, carcinogens such as aflatoxin, produced by the fungal genus Aspergillus can directly cause HCC; aflatoxin contaminates many crops in the developing world, notably cereals and nuts. Alpha-Fetoprotein is a marker that may suggest the presence of HCC. There is also evidence that the metabolic syndrome contributes to risk of developing HCC.

Cholangiocarcinoma (A) is an adenocarcinoma of the bile ducts. Risk factors for the development of cholangiocarcinoma include primary sclerosing cholangitis, parasitic liver fluke infection and exposure to medical imaging contrast media.

Alpha 1-Antitrypsin deficiency (C) results in destruction of tissues due to the lack of inhibition of neutrophil proteases, resulting in emphysema in the lungs and cirrhosis.

Haemosiderosis (D) is defined as excess haemosiderin deposition due to an acquired cause (alcohol and blood transfusions). Unless severe, there is no architectural change and therefore no development of cirrhosis.

Primary sclerosing cholangitis (H) affects the large intra- and extra- hepatic ducts. Histological features include periductal fibrosis that even- tually invades the lumen causing concentric onion-ring fibrosis.

96
Q
  1. Liver pathology
A Cholangiocarcinoma
B Cirrhosis
C Alpha1-Antitrypsin deficiency
D Haemosiderosis
E Primary biliary cirrhosis
F Haemochromatosis
G Hepatocellular carcinoma
H Primary sclerosing cholangitis 
I Wilson’s disease

5 A 45-year-old woman presents to accident and emergency with jaundice and pruritis. Xanthelasma are noted on examination. The patient is found to be anti-mitochondrial antibody positive.

A

5) E

Primary biliary cirrhosis (PBC; E) is an autoimmune disease of the liver, affecting the small and medium-sized intra-hepatic ducts. The primary histological feature is the dense accumulation of lymphocytes around bile ducts creating granulomas and total destruction of the ducts. This results in an obstructive cholestasis causing the triad of jaundice, xanthelasma (cholesterol is normally excreted in bile) and pruritis. Biochemically PBC is linked with anti-mitochondrial antibodies, as well as raised ALP, GGT, IgM and cholesterol. PBC is also strongly associated with Sjögren’s syndrome.

Cholangiocarcinoma (A) is an adenocarcinoma of the bile ducts. Risk factors for the development of cholangiocarcinoma include primary sclerosing cholangitis, parasitic liver fluke infection and exposure to medical imaging contrast media.

Alpha 1-Antitrypsin deficiency (C) results in destruction of tissues due to the lack of inhibition of neutrophil proteases, resulting in emphysema in the lungs and cirrhosis.

Haemosiderosis (D) is defined as excess haemosiderin deposition due to an acquired cause (alcohol and blood transfusions). Unless severe, there is no architectural change and therefore no development of cirrhosis.

Primary sclerosing cholangitis (H) affects the large intra- and extra- hepatic ducts. Histological features include periductal fibrosis that even- tually invades the lumen causing concentric onion-ring fibrosis.

97
Q
  1. Plaques of multiple sclerosis
    The activity of the plaques in a 25-year-old multiple sclerosis patient is described with the presence of oedema and macrophages, and some myelin breakdown. Which ICDNS (International Classification of Diseases of the Nervous System) plaque type classification best fits the description?
A Acute plaque
B Early chronic active plaque
C Late chronic active plaque
D Chronic inactive plaque
E Shadow plaque
A

B

Multiple sclerosis (MS) is the leading cause of disability in Western countries among young individuals between 20 and 40 years of age. The pathological hallmark of MS is the presence of demyelinating plaques on MRI scanning of the central nerves (spinal cord and brain). Particular things to look for are myelin loss, destruction of oligodendrocytes, and reactive astrogliosis, often with relative sparing of the axon cylinder until later stages. The number of plaques, which range from a few to several hundreds, is indicative of the severity of the disease.

According to the activity, the International Classification of Diseases classifies plaques as follows;
Acute plaque (A): Minor changes (e.g. oedema) and often difficult to recognize
Early chronic active plaque (B): Oedema and macrophages, indicative of an inflammatory disorder of the central nervous system, with some myelin breakdown. Reactive astrocytosis is present
Late chronic active plaque (C): Complete loss of myelin. Some macrophages will contain myelin debris and there will be often very mild perivascular inflammation at this stage with enlarged perivascular spaces
Chronic inactive plaque (D): Complete loss of myelin with the absence of macrophages
Shadow plaque (E): Nearly complete remyelination as a thin myelin with some scattered macrophages and a mild microglial up-regulation.

98
Q
  1. Skin pathology
A Pemphigoid
B Bowen’s disease
C Pityriasis rosea
D Lichen planus
E Actinic keratosis
F Psoriasis
G Basal cell carcinoma 
H Erythema multiforme 
I Malignant melanoma 
J Pemphigus

1 A 65-year-old man presents to his GP with blisters along his left arm that are about 1.0 cm in diameter. Gentle rubbing of the affected area does not lead to skin exfoliation.

A

1) A

Pemphigoid (A) is an autoimmune deep bullous (blisters >0.5cm) condition that occurs in the elderly. Bullae are fluid filled and therefore do not rupture easily; pemphigoid is Nikolsky sign negative. The underlying pathology involves IgG binding to hemidesmosomes. This causes activation of eosinophils that are recruited to the area. Pemphigoid should not be confused with pemphigus (option J), also an autoimmune bullous disease that affects middle-aged patients, causing superficial bullae on the skin (Nikolsky positive). IgG bind to desmosomes in the intra-epidermal region resulting in acantholysis.

Bowen’s disease (B) is also called squamous carcinoma in situ. Histological features include atypical cells involving the full thickness of the epidermis but not invading the basement membrane.

Pityriasis rosea (C) begins as a single scaly macule that is salmon-pink, <5cm and known as a ‘herald patch’. After several days multiple small pink rashes appear posteriorly creating a fir-tree pattern.

Lichen planus (D) histologically appears as inflammation at the epidermal–dermal junction. The lymphocytic infiltrate creates a saw-tooth pattern. The 7 Ps reflect the features of lichen planus: pruritic, purple, polygonal, planar, popular, plaques and pearl sheen.

Basal cell carcinomas (G) occur secondary to sun exposure. They are very invasive locally and are hence termed ‘rodent ulcers.’ On examination they are pearly, raised, irregular, ulcerated and have telangiectasia.

99
Q
  1. Skin pathology
A Pemphigoid
B Bowen’s disease
C Pityriasis rosea
D Lichen planus
E Actinic keratosis
F Psoriasis
G Basal cell carcinoma 
H Erythema multiforme 
I Malignant melanoma 
J Pemphigus

2 A 38-year-old man on the respiratory ward has been diagnosed with Mycoplasma pneumoniae and develops a number of target shaped rashes on his body.

A

2) H

Erythema multiforme (H) is a hypersensitivity reaction secondary to infections (herpes simplex, mycoplasmas and fungi) and drugs (penicillin, phenytoin and barbiturates). The pathogenesis is unclear but is thought to be due to immune-complex deposition in the microvasculature of the skin and oral mucous membranes. Most commonly the rash is self-limiting and target shaped as well as being maculo-papular with sub-epidermal bullae in the centre. In severe cases, the rash may involve mucosal surfaces leading to Steven–Johnson’s syndrome, characterized by epidermal necrosis with minimal inflammatory cell infiltrate.

Bowen’s disease (B) is also called squamous carcinoma in situ. Histological features include atypical cells involving the full thickness of the epidermis but not invading the basement membrane.

Pityriasis rosea (C) begins as a single scaly macule that is salmon-pink, <5cm and known as a ‘herald patch’. After several days multiple small pink rashes appear posteriorly creating a fir-tree pattern.

Lichen planus (D) histologically appears as inflammation at the epidermal– dermal junction. The lymphocytic infiltrate creates a saw-tooth pattern. The 7 Ps reflect the features of lichen planus: pruritic, purple, polygonal, planar, popular, plaques and pearl sheen.

Basal cell carcinomas (G) occur secondary to sun exposure. They are very invasive locally and are hence termed ‘rodent ulcers.’ On examination they are pearly, raised, irregular, ulcerated and have telangiectasia.

100
Q
  1. Skin pathology
A Pemphigoid
B Bowen’s disease
C Pityriasis rosea
D Lichen planus
E Actinic keratosis
F Psoriasis
G Basal cell carcinoma 
H Erythema multiforme 
I Malignant melanoma 
J Pemphigus

3 A 45-year-old woman presents to her GP with salmon-pink plaques with a silver–white scale on the extensor surfaces of her elbows.

A

3) F

Psoriasis (F) is an autoimmune condition primarily affecting the extensor surfaces of the skin. Histological features include parakeratosis (corneum nuclei mixed with keratin to form a thick keratin layer creating ‘silvery scales’); Munro-abscesses (white blood cells entering the corneum); loss of the granular layer leading to pin-point bleeding (Auspitz sign); clubbing of the rete ridges, whereby they grow downwards leading to a ‘test-tubes in a rack’ appearance. Clinical features include salmon-pink plaques with a silver–white scale on the skin and onycholysis.

Bowen’s disease (B) is also called squamous carcinoma in situ. Histological features include atypical cells involving the full thickness of the epidermis but not invading the basement membrane.

Pityriasis rosea (C) begins as a single scaly macule that is salmon-pink, <5cm and known as a ‘herald patch’. After several days multiple small pink rashes appear posteriorly creating a fir-tree pattern.

Lichen planus (D) histologically appears as inflammation at the epidermal– dermal junction. The lymphocytic infiltrate creates a saw-tooth pattern. The 7 Ps reflect the features of lichen planus: pruritic, purple, polygonal, planar, popular, plaques and pearl sheen.

Basal cell carcinomas (G) occur secondary to sun exposure. They are very invasive locally and are hence termed ‘rodent ulcers.’ On examination they are pearly, raised, irregular, ulcerated and have telangiectasia.

101
Q
  1. Skin pathology
A Pemphigoid
B Bowen’s disease
C Pityriasis rosea
D Lichen planus
E Actinic keratosis
F Psoriasis
G Basal cell carcinoma 
H Erythema multiforme 
I Malignant melanoma 
J Pemphigus

4 A 54-year-old man is referred to the dermatologist with a brown warty lesion on his nose which has a rough consistency. Biopsy of the lesion reveals solar elastosis.

A

4) E

Actinic keratosis (solar keratosis; E) is defined as epidermal dysplasia that occurs secondary to sunlight and presents as a brown–red warty lesion with a sandpaper-like consistency. Histological features include solar elastosis, focal parakeratosis, atypical cells and inflammatory cell infiltrates. Actinic keratosis does not affect the full thickness of the epidermis. It is a premalignant condition that may progress to squamous cell carcinoma in approximately 20% of cases.

Bowen’s disease (B) is also called squamous carcinoma in situ. Histological features include atypical cells involving the full thickness of the epidermis but not invading the basement membrane.

Pityriasis rosea (C) begins as a single scaly macule that is salmon-pink, <5cm and known as a ‘herald patch’. After several days multiple small pink rashes appear posteriorly creating a fir-tree pattern.

Lichen planus (D) histologically appears as inflammation at the epidermal– dermal junction. The lymphocytic infiltrate creates a saw-tooth pattern. The 7 Ps reflect the features of lichen planus: pruritic, purple, polygonal, planar, popular, plaques and pearl sheen.

Basal cell carcinomas (G) occur secondary to sun exposure. They are very invasive locally and are hence termed ‘rodent ulcers.’ On examination they are pearly, raised, irregular, ulcerated and have telangiectasia.

102
Q
  1. Skin pathology
A Pemphigoid
B Bowen’s disease
C Pityriasis rosea
D Lichen planus
E Actinic keratosis
F Psoriasis
G Basal cell carcinoma 
H Erythema multiforme 
I Malignant melanoma 
J Pemphigus

5 A 59-year-old woman presents to her dermatologist with a 3 cm black irregular lesion on her cheek. Over the next month the lesion spreads to cover 6 cm with new onset pain.

A

5) I

Malignant melanoma (I) is a malignant tumour of melanocytes. The characteristic features can be remembered by the mnemonic ABCDE: asymmetry, border irregularity, colour (usually black; sometimes demonstrate colours of the French flag), diameter >5cm and evolution (change in size, colour and/or new onset itchiness/pain). Malignant melanomas initially grow radially in situ within the epidermis; over time there is growth vertically into the dermis, eventually leading to metastases. Sub-types include lentigomaligna (LM), acrallentigious (AL), superficial spreading (SS) and nodular (N).

Bowen’s disease (B) is also called squamous carcinoma in situ. Histological features include atypical cells involving the full thickness of the epidermis but not invading the basement membrane.

Pityriasis rosea (C) begins as a single scaly macule that is salmon-pink, <5cm and known as a ‘herald patch’. After several days multiple small pink rashes appear posteriorly creating a fir-tree pattern.

Lichen planus (D) histologically appears as inflammation at the epidermal– dermal junction. The lymphocytic infiltrate creates a saw-tooth pattern. The 7 Ps reflect the features of lichen planus: pruritic, purple, polygonal, planar, popular, plaques and pearl sheen.

Basal cell carcinomas (G) occur secondary to sun exposure. They are very invasive locally and are hence termed ‘rodent ulcers.’ On examination they are pearly, raised, irregular, ulcerated and have telangiectasia.

103
Q
  1. Causes of dementia
    A 72-year-old woman is diagnosed with a disease that accounts for 50–75% of all cases of dementia. The four characteristic pathological features for her diagnosis are severe brain atrophy, loss of neurons, senile plaques and neuro-fibllirary tangles. What is the most likely diagnosis?
A Huntington’s disease
B Alzheimer’s disease
C Multiple system atrophy
D Dementia with Lewy bodies
E Parkinson’s disease
A

B

Alzheimer’s disease (AD) (B) accounts for 50–75% of all cases of dementia in Western countries. Dementia is the progressive loss of cognitive function due to degeneration of the cerebral cortex. There is severe brain atrophy particularly prominent in the hippocampus and the frontal lobes and the brain weight is reduced to 1000 grams or less (normal average being 1400 grams). Histological hallmarks of the disease include senile plaques, which are complex spherical structures involving the grey matter and the aggregation of beta-amyloid appears to play a central role in developing the senile plaques. Neurofibrillary tangles are abnormal tangles in neuronal cell bodies of insoluble cytoskeletal-like tau protein. It is believed that the major antigenic component is the phosphorylated tau.

Dementia with Lewy bodies (D) is another likely possibility for this patient and even though it can only be diagnosed at autopsy, cortical Lewy bodies will be prominent. This condition is believed to account for 25% of all dementias.

The remaining three diseases (A, C, E) are not often associated with dementia. Huntington’s disease characteristically shows cerebral atrophy in the caudate nucleus and putamen and several changes in neurotransmitters. Multiple system atrophy is associated with glial. cytoplasmic inclusion bodies (i.e. Papp–Lantos bodies). Parkinson’s disease is characteristically associated with depigmentation of the substantia nigra.

104
Q
  1. Renal pathology
A Nephritic syndrome
B Wegener’s granulomatosis
C Membranous glomerulonephritis
D Acute tubular necrosis
E Minimal change glomerulo-nephritis
F Goodpasture’s syndrome
G IgA nephropathy
H Nephrotic syndrome
I Focal segmental glomerulo-nephritis

1 A 45-year-old man presents to accident and emergency with haematuria and admits to passing less urine than previously. He is found to be hypertensive. Microscopy of the patient’s urine reveals red and white cell casts.

A

1) A

Nephritic syndrome (A) involves the following: haematuria, red cell and white cell casts, dysmorphic red cells, oliguria and hypertension. The pathogenesis begins with inflammation of glomerular vessels allowing red blood cells to enter the renal tubule; as they enter they are damaged. The body compensates for the inflammation by slowing renal blood flow causing oliguria, which leads to water retention and hence hypertension. Cellular casts form as a result of Tamm–Horsefall secretions in the distal collecting duct and collecting duct that ‘glue’ cells together, hence forming a cast.

Wegener’s granulamatosis (B) is a c-ANCA mediated pauci-immune RPGN, creating an absent/scant pattern on IgG and C3 immunofluorescence staining.
Membranous glomerulonephritis (C) is defined on histological investigation as a thickened glomerular basement membrane, spike/dome protrusions, sub-epithelial immunoglobulin and a granular staining pattern.

Minimal change glomerulonephritis (E) appears normal with light microscopy but podocyte effacement is visible on electron microscopy. Most cases will respond to steroid treatment.

Focal segmental glomerulonephritis (I) appears with obliterated lumen and podocyte effacement on histological examination. Focal segmental glomerulonephritis is not responsive to steroids.